You are on page 1of 103

Adult spine self-scored self-assessment examination AAOS 2015

CLINICAL SITUATION FOR QUESTIONS 1 THROUGH 3

A B

C D

Figures 1a through 1d are the contrast-enhanced MR images of a 56-year-old


man with a 2-week history of increasingly severe back pain. He denies any
injuries to his back, fevers, or chills.

Question 1 of 100

1
Adult spine self-scored self-assessment examination AAOS 2015

What is the most likely diagnosis?

1- Degenerative disk disease


2- Disk herniation with spinal stenosis
3- Epidural and paraspinal abscess
4- Muscle strain

PREFERRED RESPONSE: 3- Epidural and paraspinal abscess

Question 2 of 100

Patients initially treated with intravenous (IV) antibiotics are at higher risk for
failure of nonsurgical treatment in the setting of

1- obesity.
2- diabetes.
3- abscess extending over 3 vertebrae.
4- blood culture findings positive for coagulase-negative Staphylococci.

PREFERRED RESPONSE: 2- diabetes.

Question 3 of 100

Which clinical finding most strongly suggests that nonsurgical care should be
discontinued and surgical intervention is necessary?

1- Progressive weakness
2- C-reactive protein (CRP) level of 75
3- Increased low-back pain
4- White blood cell (WBC) count of 11

PREFERRED RESPONSE: 1- Progressive weakness

DISCUSSION

2
Adult spine self-scored self-assessment examination AAOS 2015

Epidural abscesses are potentially devastating. Nonsurgical care may be


chosen for select patients. A baseline failure rate of 8.3% increases based on
patient risk factors, which include a history of IV drug abuse, diabetes, age
older than 65, CRP level higher than 115, WBC level higher than 12.5, and
Staphylococcus aureus as the causative organism. Immunosuppression and
abscess size are not significant risk factors for failure of nonsurgical care.
Nonsurgical care may be regarded as "failed" if there is worsening of a
patient's neurologic status. When nonsurgical care fails, delayed surgery is
less successful at restoring motor function (vs early surgery).

RECOMMENDED READINGS

Kim SD, Melikian R, Ju KL, Zurakowski D, Wood KB, Bono CM, Harris MB. Independent
predictors of failure of nonoperative management of spinal epidural abscesses. Spine J. 2014
Aug 1;14(8):1673-9. doi: 10.1016/j.spinee.2013.10.011. Epub 2013 Oct 30. PubMed PMID:
24373683. View Abstract at PubMed
Patel AR, Alton TB, Bransford RJ, Lee MJ, Bellabarba CB, Chapman JR. Spinal epidural
abscesses: risk factors, medical versus surgical management, a retrospective review of 128
cases. Spine J. 2014 Feb 1;14(2):326-30. doi: 10.1016/j.spinee.2013.10.046. Epub 2013
Nov 12. PubMed PMID: 24231778. View Abstract at PubMed
Duarte RM, Vaccaro AR. Spinal infection: state of the art and management algorithm. Eur
Spine J. 2013 Dec;22(12):2787-99. doi: 10.1007/s00586-013-2850-1. Epub 2013 Jun 12.
Review. PubMed PMID: 23756630. View Abstract at PubMed

Question 4 of 100

Which factor is most important when attempting to prevent interbody graft


subsidence?

1- End plate burring


2- Surface contact area
3- Bone quality
4- Use of rigid fixation

PREFERRED RESPONSE: 3- Bone quality

DISCUSSION

3
Adult spine self-scored self-assessment examination AAOS 2015

Osteoporosis can affect all aspects of spinal stability and is the most critical
factor regarding spinal implant failure. Burring of the end plates may decrease
strength of the interface with the uncovering of "softer" cancellous bone.
Increasing the surface contact area may help prevent subsidence but is not
as important as bone quality. Stress shielding through rigid fixation may lead
to construct failure.

RECOMMENDED READINGS

Benzel E (ed): Biomechanics of Spine Stabilization. Rolling Meadows, IL, American Association
of Neurological Surgeons, 2001, pp 446-447.
Goldhahn J, Reinhold M, Stauber M, Knop C, Frei R, Schneider E, Linke B. Improved anchorage
in osteoporotic vertebrae with new implant designs. J Orthop Res. 2006 May;24(5):917-25.
PubMed PMID: 16583445. View Abstract at PubMed

Question 5 of 100

Figure 5 is a T2-weighted MR image of a 26-year-old man who has had left


leg pain for 3 months that has failed nonsurgical treatment. Surgical
decompression is planned. Which approach would provide the most direct
ability to perform surgical decompression?

1- Posterior midline approach


2- Retroperitoneal approach
3- Far lateral approach
4- Transpsoas approach

4
Adult spine self-scored self-assessment examination AAOS 2015

PREFERRED RESPONSE: 3- Far lateral approach

DISCUSSION

The MR image shows a far lateral disk herniation impinging on the exiting
nerve root lateral to the exiting foramen. This is reached most directly with a
far lateral (Wiltse) approach. This is a posterior paramedian approach that
uses the interval between the paraspinal muscles (multifidus and longissimus)
and arrives onto the facet joints. The intertransverse membrane can then be
released, exposing the far lateral disk herniation. A posterior midline approach
will allow easy access to the spinal canal, which is medial to the disk
herniation, and will not allow for easy disk removal without the need for a
facetectomy, which would destabilize the level. An anterior approach would
not allow for access to the far lateral disk herniation, nor would a traditional
retroperitoneal or newer transpsoas approach.

RECOMMENDED READINGS

Wiltse LL, Spencer CW. New uses and refinements of the paraspinal approach to the lumbar
spine. Spine (Phila Pa 1976). 1988 Jun;13(6):696-706. PubMed PMID: 3175760. View
Abstract at PubMed
Epstein NE. Evaluation of varied surgical approaches used in the management of 170 far-
lateral lumbar disc herniations: indications and results. J Neurosurg. 1995 Oct;83(4):648-56.
PubMed PMID: 7674015. View Abstract at PubMed

Question 6 of 100

Which intervention most effectively prevents surgical-site infections following


spine surgery?

1- Bathing the day of surgery


2- Intravenous (IV) vancomycin
3- Preincision IV antibiotics
4- Vancomycin powder in wound

PREFERRED RESPONSE: 3- Preincision IV antibiotics

DISCUSSION

5
Adult spine self-scored self-assessment examination AAOS 2015

The use of IV antibiotics for prophylaxis of surgical-site infection is supported


by Level 1 evidence in spine surgery. It has been given a "B" recommendation
by the North American Spine Society. The use of specific bathing solutions the
day of surgery may be beneficial, but the evidence in spine surgery is lacking.
Similarly, evidence for use of vancomycin (either topically or IV) is not
supported by high-level studies, although retrospective and basic science
studies support topical vancomycin use.

RECOMMENDED READINGS

Brown MD, Brookfield KF. A randomized study of closed wound suction drainage for extensive
lumbar spine surgery. Spine (Phila Pa 1976). 2004 May 15;29(10):1066-8. PubMed PMID:
15131430. View Abstract at PubMed
Diab M, Smucny M, Dormans JP, Erickson MA, Ibrahim K, Lenke LG, Sucato DJ, Sanders JO.
Use and outcomes of wound drain in spinal fusion for adolescent idiopathic scoliosis. Spine
(Phila Pa 1976). 2012 May 15;37(11):966-73. doi: 10.1097/BRS.0b013e31823bbf0b.
PubMed PMID: 22037527. View Abstract at PubMed
Evaniew N, Khan M, Drew B, Peterson D, Bhandari M, Ghert M. Intrawound vancomycin to
prevent infections after spine surgery: a systematic review and meta-analysis. Eur Spine J.
2014 May 18. [Epub ahead of print] PubMed PMID: 24838506. View Abstract at PubMed
Rubinstein E, Findler G, Amit P, Shaked I. Perioperative prophylactic cephazolin in spinal
surgery. A double-blind placebo-controlled trial. J Bone Joint Surg Br. 1994 Jan;76(1):99-
102. PubMed PMID: 8300691. View Abstract at PubMed
Savage JW, Anderson PA. An update on modifiable factors to reduce the risk of surgical site
infections. Spine J. 2013 Sep;13(9):1017-29. doi:10.1016/j.spinee.2013.03.051. Epub 2013
May 24. Review. PubMed PMID: 23711958. View Abstract at PubMed
Shaffer WO, Baisden JL, Fernand R, Matz PG; North American Spine Society. An evidence-
based clinical guideline for antibiotic prophylaxis in spine surgery. Spine J. 2013
Oct;13(10):1387-92. doi: 10.1016/j.spinee.2013.06.030. Epub 2013 Aug 27. Review.
PubMed PMID: 23988461. View Abstract at PubMed

Question 7 of 100

Figures 7a through 7d are the images of a 31-year-old obese woman who has
a long history of low-back pain and intermittent bilateral lower extremity pain.
Five days ago her symptoms increased markedly and she was given pain
medications upon presentation to her primary care physician. Three days ago
she noticed that her bed was wet upon awakening; she also had numbness
and tingling in her peroneal area and lower extremities and weakness in her
lower extremities. She is brought to your office in a wheelchair. Her
examination reveals diminished sensation to light touch in the L4 to S4
dermatomes and 0-1/5 strength in all muscle groups in her bilateral lower

6
Adult spine self-scored self-assessment examination AAOS 2015

extremities with the exception of her hip flexors, hip adductors, and
quadriceps, which are 5/5 in strength. She has decreased sphincter tone on
rectal examination. You recommend immediate decompressive laminectomy.
What is the likelihood she will regain bladder function after surgery?

A B

C D

1- 0% to 20%
2- 21% to 40%
3- 41% to 60%
4- 61% to 80%

7
Adult spine self-scored self-assessment examination AAOS 2015

PREFERRED RESPONSE: 2- 21% to 40%

DISCUSSION

This patient has congenital and acquired spinal stenosis with multilevel disk
protrusions that have both chronic (calcified) and acute components, resulting
is multilevel cauda equina compression and acute cauda equina syndrome of
more than 48 hours duration. Most studies indicate that patients who undergo
decompression within 48 hours of symptom onset have a better prognosis for
neurologic recovery than those who undergo decompression after 48 hours.
Among patients with urinary incontinence, 1 study indicated that 43%
remained incontinent at follow-up visits, but this study included a mix of early
and late surgical patients. In another series of 44 patients with acute cauda
equina syndrome, chronic bowel and bladder dysfunction were issues for 63%
of those for whom surgery was delayed for more than 48 hours after symptom
onset.

RECOMMENDED READINGS

McCarthy MJ, Aylott CE, Grevitt MP, Hegarty J. Cauda equina syndrome: factors affecting
long-term functional and sphincteric outcome. Spine (Phila Pa 1976). 2007 Jan 15;32(2):207-
16. PubMed PMID: 17224816. View Abstract at PubMed
Ahn UM, Ahn NU, Buchowski JM, Garrett ES, Sieber AN, Kostuik JP. Cauda equina syndrome
secondary to lumbar disc herniation: a meta-analysis of surgical outcomes. Spine (Phila Pa
1976). 2000 Jun 15;25(12):1515-22. PubMed PMID: 10851100. View Abstract at PubMed
Shapiro S. Medical realities of cauda equina syndrome secondary to lumbar disc herniation.
Spine (Phila Pa 1976). 2000 Feb 1;25(3):348-51; discussion 352. PubMed PMID: 10703108.
View Abstract at PubMed

Question 8 of 100

Among patients with adolescent idiopathic scoliosis, a thoracolumbosacral


orthosis is most effective for which type of curve?

1- Apex of the curve is at T3


2- Apex of the curve is at T8
3- Apex of the curve is at L1
4- Apex of the curve is at L2

8
Adult spine self-scored self-assessment examination AAOS 2015

PREFERRED RESPONSE: 2- Apex of the curve is at T8

DISCUSSION

A thoracolumbosacral orthosis is most effective for bracing of curves when the


apex is at T7 or below. Bracing is used for patients who are skeletally
immature (Risser stage 0, 1, or 2), and it is recommended that the brace be
worn 16 to 23 hours per day and continued until skeletal maturity or until the
curve progresses to beyond 45 degrees, at which point bracing is no longer
considered effective.

RECOMMENDED READINGS

Luhmann SJ, Skaggs DL: Pediatric spine conditions, in Lieberman JR (ed):


AAOS Comprehensive Orthopaedic Review. Rosemont, IL, American Academy of Orthopaedic
Surgeons, 2009, pp 245-265.
Shaughnessy WJ. Advances in scoliosis brace treatment for adolescent idiopathic scoliosis.
Orthop Clin North Am. 2007 Oct;38(4):469-75, v. Review. PubMed PMID: 17945126.
View Abstract at PubMed

Question 9 of 100

Figures 9a through 9c are the preoperative radiographs and a T2-weighted


MR image of a patient treated with surgery for spondylolisthesis and
neuroforaminal stenosis. Figure 9d is the postsurgical radiograph. Interbody
fusion offers which advantage over posterolateral fusion (PLF)?

A B

9
Adult spine self-scored self-assessment examination AAOS 2015

C D

1- Decreased blood loss


2- Less neural retraction
3- Improved neuroforaminal height
4- Lower fusion rates

PREFERRED RESPONSE: 3- Improved neuroforaminal height

DISCUSSION

Interbody fusion, when compared to PLF, is a predictor of more substantial


blood loss. Multilevel posterior lumbar interbody fusion (PLIF) is an
independent predictor of blood loss for posterior spine fusion. Some
retrospective studies suggest that fusion rates are higher for transforaminal
lumbar interbody fusion (TLIF) than PLF, but this finding has not been borne
out in prospective studies. The main advantage of TLIF in the context of this
question is restoration of neuroforaminal height, and many surgeons will
consider TLIF or PLIF for that reason. The parasagittal MR image seen in Figure
9c shows neuroforaminal narrowing. The pre- and postsurgical radiographs
show a difference in neuroforaminal height.

RECOMMENDED READINGS

DiPaola CP, Molinari RW. Posterior lumbar interbody fusion. J Am Acad Orthop Surg. 2008
Mar;16(3):130-9. Review. PubMed PMID: 18316711. View Abstract at PubMed

10
Adult spine self-scored self-assessment examination AAOS 2015

Eismont FJ, Norton RP, Hirsch BP. Surgical management of lumbar degenerative
spondylolisthesis. J Am Acad Orthop Surg. 2014 Apr;22(4):203-13. doi: 10.5435/JAAOS-22-
04-203. Review. PubMed PMID: 24668350. View Abstract at PubMed
McAfee PC, DeVine JG, Chaput CD, Prybis BG, Fedder IL, Cunningham BW, Farrell DJ, Hess
SJ, Vigna FE. The indications for interbody fusion cages in the treatment of spondylolisthesis:
analysis of 120 cases. Spine (Phila Pa 1976). 2005 Mar 15;30(6 Suppl):S60-5. PubMed PMID:
15767888. View Abstract at PubMed

RESPONSES FOR QUESTIONS 10 THROUGH 14

1- Postoperative deep surgical site infection


2- Proximal junctional kyphosis
3- Pedicle screw cut-out
4- Pseudarthrosis
5- Sagittal imbalance
Please match the scenario described below with the most likely
complication listed above.

Question 10 of 100

A 55-year-old man who has had multiple lumbar fusions during the last 10
years and a lumbar lordosis of 25 degrees notes difficulty standing upright
without flexing his knees.

1- Postoperative deep surgical site infection


2- Proximal junctional kyphosis
3- Pedicle screw cut-out
4- Pseudarthrosis
5- Sagittal imbalance

PREFERRED RESPONSE: 5- Sagittal imbalance

Question 11 of 100

An 80-year-old woman with osteoporosis and thoracic kyphosis has a spinal


fusion from T12-S1.

11
Adult spine self-scored self-assessment examination AAOS 2015

1- Postoperative deep surgical site infection


2- Proximal junctional kyphosis
3- Pedicle screw cut-out
4- Pseudarthrosis
5- Sagittal imbalance

PREFERRED RESPONSE: 3- Pedicle screw cut-

Question 12 of 100

A 65-year-old woman with diabetes has fever and erythema 2 weeks after
undergoing instrumented spinal fusion.

1- Postoperative deep surgical site infection


2- Proximal junctional kyphosis
3- Pedicle screw cut-out
4- Pseudarthrosis
5- Sagittal imbalance

PREFERRED RESPONSE: 1- Postoperative deep surgical site infection

Question 13 of 100

A 60-year-old man with a pelvic incidence of 75 degrees undergoes an


instrumented fusion for degenerative scoliosis from T11-S1, achieving a
lumbar lordosis of 50 degrees.

1- Postoperative deep surgical site infection


2- Proximal junctional kyphosis
3- Pedicle screw cut-out
4- Pseudarthrosis
5- Sagittal imbalance

PREFERRED RESPONSE: 2- Proximal junctional kyphosis

Question 14 of 100

12
Adult spine self-scored self-assessment examination AAOS 2015

A 50-year-old woman undergoes an L4-S1 laminectomy and noninstrumented


fusion for degenerative spondylolisthesis.

1- Postoperative deep surgical site infection


2- Proximal junctional kyphosis
3- Pedicle screw cut-out
4- Pseudarthrosis
5- Sagittal imbalance
PREFERRED RESPONSE: 4- Pseudarthrosis

DISCUSSION

Complications are numerous in adult spinal deformity surgery. Many


complications are related to the patient's sagittal balance following surgery
and recognition of the potential to develop sagittal imbalance or flat-back
syndrome following spinal fusion. The quality of bone density is important in
spinal instrumented fusions, especially among older patients. Patients with
osteopenia or osteoporosis have a higher incidence of proximal-level screw
cut-out through the vertebral body into the cephalad disk space.

Proximal junctional kyphosis is common in longer instrumented fusions,


especially when instrumented to the sacrum/pelvis; when the spine is fixed in
a "flat" or hypolordotic position; when the thoracic spine is hyperkyphotic (ie,
Scheuermann kyphosis); when the end instrumented vertebrae is kyphotic;
or when the sagittal plumb line (measured from C7) is more than 4 cm forward
of the posterior corner of the sacrum.
Sagittal imbalance is a common complication when the spine is instrumented
in a hypolordotic position. This can occur with degenerative conditions that
necessitate multilevel fusions or fusions to sacrum without recognition of the
degree of lordosis the patient should have. Pelvic incidence (PI) is a
spinopelvic measurement that is a constant that measures an angle from the
hips to the midpoint of the sacral end plate. PI correlates to the amount of
lumbar lordosis that a patient would typically have in an upright position (+/-
10 degrees). If a patient has significant sagittal imbalance, he or she will have
a forward lean and lack the ability to extend the spine to stand upright. In an
attempt to stand upright, the patient may bend his or her knees or hips in a
crouched position. When extending their knees, they again lean forward.
Pseudarthrosis is common with noninstrumented fusions. Deep surgical-site
infections are uncommon but can be major complications that are difficult to
treat, necessitating formal irrigation and debridement and long-term
antibiotics. Patients with diabetes have a higher incidence of infection.

13
Adult spine self-scored self-assessment examination AAOS 2015

RECOMMENDED READINGS

Glassman SD, Bridwell K, Dimar JR, Horton W, Berven S, Schwab F. The impact of positive
sagittal balance in adult spinal deformity. Spine (Phila Pa 1976). 2005 Sep 15;30(18):2024-
9. PubMed PMID: 16166889.View Abstract at PubMed
Kim YJ, Bridwell KH, Lenke LG, Rhim S, Cheh G. Sagittal thoracic decompensation following
long adult lumbar spinal instrumentation and fusion to L5 or S1: causes, prevalence, and risk
factor analysis. Spine (Phila Pa 1976). 2006 Sep 15;31(20):2359-66. PubMed PMID:
16985465.View Abstract at PubMed

Question 15 of 100

A 25-year-old athletic woman has a 16-week history of left lower-extremity


radiating pain in an S1 distribution. MR images obtained by her family
physician reveal a large L5-S1 paracentral disk herniation impinging on the
left S1 nerve root. You suggest a left-sided L5-S1 microdiskectomy and tell
her that when comparing tubular diskectomy and open procedures

1- there are no differences in functional outcome.


2- open diskectomy is associated with superior functional outcomes.
3- tubular diskectomy is associated with superior short- and long-term
results.
4- tubular diskectomy is associated with inferior short-term results but
superior long-term outcomes.

PREFERRED RESPONSE: 1- there are no differences in functional outcome.

DISCUSSION

Several comparative studies have reported no difference in functional


outcomes between tubular diskectomy and microsurgical lumbar diskectomy.
A recent systematic review by Kamper and associates in which conventional
microdiskectomy and minimally invasive approaches were compared revealed
that there was no difference between the procedures in terms of clinical
outcomes, complication risk, or rate of revision surgery.

RECOMMENDED READINGS

14
Adult spine self-scored self-assessment examination AAOS 2015

Kamper SJ, Ostelo RW, Rubinstein SM, Nellensteijn JM, Peul WC, Arts MP, van Tulder MW.
Minimally invasive surgery for lumbar disc herniation: a systematic review and meta-analysis.
Eur Spine J. 2014 May;23(5):1021-43. doi: 10.1007/s00586-013-3161-2. Epub 2014 Jan 18.
PubMed PMID: 24442183.View Abstract at PubMed
Dasenbrock HH, Juraschek SP, Schultz LR, Witham TF, Sciubba DM, Wolinsky JP, Gokaslan
ZL, Bydon A. The efficacy of minimally invasive discectomy compared with open discectomy:
a meta-analysis of prospective randomized controlled trials. J Neurosurg Spine. 2012
May;16(5):452-62. doi: 10.3171/2012.1.SPINE11404. Epub 2012 Mar 9. PubMed PMID:
22404142.View Abstract at PubMed

Question 16 of 100

A 58-year-old man has had increasing midback pain for 8 weeks. Radiographs
reveal mild osteopenia and mild disk degeneration but no fractures or lesions.
An MRI of the spine reveals diskitis with a small-intensity signal within the
spinal canal that is consistent with an epidural abscess at T11-12. The patient
is neurologically intact but in significant pain. CT-guided biopsy of the disk
space is positive for methicillin-sensitive Staphylococcus aureus. What is the
most appropriate treatment?

1- Intravenous (IV) antibiotics for 6 weeks and clinical observation


2- Observation and bracing alone
3- Laminectomy and posterior spinal fusion with IV antibiotics
4- Anterior spinal debridement and fusion with IV antibiotics

PREFERRED RESPONSE: 1- Intravenous (IV) antibiotics for 6 weeks and


clinical observation

DISCUSSION

The treatment of spinal infections is variable. A diskitis in a patient with a


mechanically stable spine without neurologic compromise is typically treated
with needle biopsy/culture and appropriate IV antibiotics. Epidural abscess
often is considered one of the true orthopaedic emergencies that necessitates
surgical intervention. However, there is growing evidence that medical
management can be appropriate to treat epidural abscesses in certain cases.
In cases involving neurologic deterioration, surgical decompression, drainage,
and systemic IV antibiotics is the treatment of choice. Medical management
of spinal abscesses can be considered when a patient refuses surgical
decompression; surgery is contraindicated because of high risk, pain, or

15
Adult spine self-scored self-assessment examination AAOS 2015

infection; or paralysis lasting longer than 24 to 36 hours results in a likely


inability to reverse the paralysis. Patients who are neurologically intact may
also be treated with medical management alone if they are stable and have
an identifiable microorganism that can be observed closely to assess for
neurologic deterioration. If neurologic changes are noted, surgical
decompression and debridement and continued IV antibiotic therapy are
appropriate.

RECOMMENDED READINGS

Darouiche RO. Spinal epidural abscess. N Engl J Med. 2006 Nov 9;355(19):2012-20. Review.
PubMed PMID: 17093252.View Abstract at PubMed
Kim SD, Melikian R, Ju KL, Zurakowski D, Wood KB, Bono CM, Harris MB. Independent
predictors of failure of nonoperative management of spinal epidural abscesses. Spine J. 2014
Aug 1;14(8):1673-9. doi: 10.1016/j.spinee.2013.10.011. Epub 2013 Oct 30. PubMed PMID:
24373683.View Abstract at PubMed
Patel AR, Alton TB, Bransford RJ, Lee MJ, Bellabarba CB, Chapman JR. Spinal epidural
abscesses: risk factors, medical versus surgical management, a retrospective review of 128
cases. Spine J. 2014 Feb 1;14(2):326-30. doi: 10.1016/j.spinee.2013.10.046. Epub 2013
Nov 12. Review. PubMed PMID: 24231778.View Abstract at PubMed
Siddiq F, Chowfin A, Tight R, Sahmoun AE, Smego RA Jr. Medical vs surgical management of
spinal epidural abscess. Arch Intern Med. 2004 Dec 13-27;164(22):2409-12. PubMed PMID:
15596629. View Abstract at PubMed

Question 17 of 100

While performing long fusion with osteotomies for a patient with adult scoliosis
and sagittal plane deformity, the neurophysiologist reports a change in motor-
evoked potentials in the lower extremities. What is the most appropriate next
step?

1- Perform an immediate wake-up test


2- Continue with the surgery and reassess in 5 minutes
3- Administer intravenous methylprednisolone
4- Ensure that mean arterial blood pressure is 80 or higher

PREFERRED RESPONSE: 4- Ensure that mean arterial blood pressure is 80 or


higher

DISCUSSION

16
Adult spine self-scored self-assessment examination AAOS 2015

The use of intraoperative neuromonitoring is expanding, especially in the


setting of deformity surgery. Changes in monitoring are concerning for the
possibility of a neurologic injury; however, several other factors can alter
signals. These include hypotension, changes in anesthesia depth and
medications, the use of paralytic agents, and technical issues such as leads
falling out or becoming disconnected. If a change in neuromonitoring signals
is noted, these factors should be checked first to rule out false-positive
findings. If this does not correct the problem, the wound should be explored
to ensure there is no compression on the neural elements. Finally, if the
deformity has been corrected, some of the correction can be released in an
attempt to improve the signals. A wake-up test is difficult to perform and does
not provide specific information regarding the location of the problem or how
to correct it. Steroids may be used depending on surgeon preference, but
should not be used until neurologic injury is ruled out. Any neuromonitoring
changes always necessitate immediate investigation.

RECOMMENDED READINGS

Ziewacz JE, Berven SH, Mummaneni VP, Tu TH, Akinbo OC, Lyon R, Mummaneni PV. The
design, development, and implementation of a checklist for intraoperative neuromonitoring
changes. Neurosurg Focus. 2012 Nov;33(5):E11. doi: 10.3171/2012.9.FOCUS12263. PubMed
PMID: 23116091. View Abstract at PubMed
Malhotra NR, Shaffrey CI. Intraoperative electrophysiological monitoring in spine surgery.
Spine (Phila Pa 1976). 2010 Dec 1;35(25):2167-79. doi: 10.1097/BRS.0b013e3181f6f0d0.
Review. PubMed PMID: 21102290. View Abstract at PubMed

Question 18 of 100

A B

17
Adult spine self-scored self-assessment examination AAOS 2015

Figures 18a and 18b are the lumbar spine radiographs of a 72-year-old man
with no significant medical history who has had severe back pain for 3 weeks.
He denies radiating symptoms, weakness, or numbness when he is seen in
the emergency department. He is sent home with a soft corset. At his follow-
up visit he continues to describe significant back pain with activity that is not
relieved with oral narcotic mediations. A follow-up CT scan shows a
nondisplaced fracture through all 3 columns of the spine. What is the most
appropriate treatment?

1- Nonsteroidal anti-inflammatory drugs (NSAIDS), physical therapy, and


activity modification
2- Continued soft corset use
3- Thoracolumbosacral orthosis (TLSO) bracing
4- Posterior stabilization

PREFERRED RESPONSE: 4- Posterior stabilization

DISCUSSION

The patient has previously undiagnosed ankylosing spondylitis. Radiographs


reveal nonmarginal syndesmophytes throughout the lumbar spine. The CT
scan reveals a nondisplaced 3-column fracture. Many patients with missed
spinal injuries present in followup with neurologic worsening or progressive
deformity. Fractures can often involve all 3 columns, including the posterior
elements. In patients with ankylosing spondylitis, this represents an unstable
injury and a high likelihood of displacement with nonsurgical treatment.
Surgical treatment in the form of a posterior spinal fusion is indicated. Because
the fracture is nondisplaced and the patient is neurologically intact,
decompression via an anterior approach is not indicated. Bracing either with
a TLSO brace or a soft corset will not provide sufficient stability for this fracture
pattern. Physical therapy and NSAIDS are not indicated in this scenario.

RECOMMENDED READINGS

Hu SS, Ananthakrishnan D. Ankylosing spondylitis. In: Herkowitz HN, Garfin SR, eds. The Spine. 5th ed.
Philadelphia, PA: Elsevier; 2006:763-761.

Blam OG, Cotler JM: Fractures in the stiff and osteoporotic spine. In: Browner BD, Jupiter JB, Levine
AM, Trafton PG, eds. Skeletal Trauma: Basic Science, Management, and Reconstruction,. 3rd ed.
Philadelphia, PA: Elsevier; 2003.

18
Adult spine self-scored self-assessment examination AAOS 2015

Question 19 of 100

A B

Figures 19a and 19b are the CT scans of an 18-year-old man who was a
restrained driver in a rollover motor vehicle collision. What was the primary
mechanism of injury?

1- Axial load
2- Rotation
3- Extension
4- Distraction

PREFERRED RESPONSE: 4- Distraction

DISCUSSION

This injury pattern represents a flexion distraction injury, or a Chance fracture.


The imaging hallmark of this injury is the axial split of the pedicle seen on the
sagittal CT scan. This injury necessitates surgical intervention to prevent
progressive deformity and pain. A rotation mechanism results in a dislocation
with a rotational deformity at the level of injury. An extension force causes
failure of the anterior column in tension with compression of the posterior
column. An axial load force would result in a burst fracture.

19
Adult spine self-scored self-assessment examination AAOS 2015

RECOMMENDED READINGS

Bono CM, Rinaldi MD. Thoracolumbar trauma. In: Spivak JM, Connolly PJ, eds. Orthopaedic
Knowledge Update: Spine 3. Rosemont, IL: American Academy of Orthopaedic Surgeons;
2006:201-216.
Vaccaro AR, Baron EM, Sanfilippo J, Jacoby S, Steuve J, Grossman E, DiPaola M, Ranier P,
Austin L, Ropiak R, Ciminello M, Okafor C, Eichenbaum M, Rapuri V, Smith E, Orozco F, Ugolini
P, Fletcher M, Minnich J, Goldberg G, Wilsey J, Lee JY, Lim MR, Burns A, Marino R, DiPaola C,
Zeiller L, Zeiler SC, Harrop J, Anderson DG, Albert TJ, Hilibrand AS. Reliability of a novel
classification system for thoracolumbar injuries: the Thoracolumbar Injury Severity Score.
Spine (Phila Pa 1976). 2006 May 15;31(11 Suppl):S62-9; discussion S104. PubMed
PMID:16685239.View Abstract at PubMed

Question 20 of 100

A number of potential complications are associated with the direct lateral


approach to the lumbar spine; which complication is most common?

1- Infection
2- Iliopsoas weakness
3- Injury to the aorta
4- Foot drop

PREFERRED RESPONSE: 2- Iliopsoas weakness

DISCUSSION

Sofianos and associates examined the cases of 45 patients who underwent


the lateral transpsoas approach and found that 18 of 45 patients (40%)
experienced at least 1 complication. The most common complication was
postsurgical weakness of the iliopsoas, which was an issue for 10 of 45
patients (22.2%). The second-most-common complication in this series was
anterior thigh hypoesthesia. This occurred in 8 of 45 patients (17.8%). A
series of 600 patients by Rodgers and associates noted that thigh pain and
psoas weakness following a direct lateral approach to the lumbar spine were
both "nearly universal" but almost "always transient."

RECOMMENDED READINGS

20
Adult spine self-scored self-assessment examination AAOS 2015

Sofianos DA, Briseño MR, Abrams J, Patel AA. Complications of the lateral transpsoas
approach for lumbar interbody arthrodesis: a case series and literature review. Clin Orthop
Relat Res. 2012 Jun;470(6):1621-32. doi: 10.1007/s11999-011-2088-3. Review. PubMed
PMID: 21948287.View Abstract at PubMed
Lee YP, Regev GJ, Chan J, Zhang B, Taylor W, Kim CW, Garfin SR. Evaluation of hip flexion
strength following lateral lumbar interbody fusion. Spine J. 2013 Oct;13(10):1259-62. doi:
10.1016/j.spinee.2013.05.031. Epub 2013 Jul 12. PubMed PMID: 23856656. View Abstract
at PubMed
Rodgers WB, Gerber EJ, Patterson J. Intraoperative and early postoperative complications in
extreme lateral interbody fusion: an analysis of 600 cases. Spine (Phila Pa 1976). 2011 Jan
1;36(1):26-32. doi: 10.1097/BRS.0b013e3181e1040a. PubMed PMID: 21192221. View
Abstract at PubMed

CLINICAL SITUATION FOR QUESTIONS 21 AND 22

Figures 21a through 21c are the preoperative lateral standing radiograph,
axial T2-weighted MR image at L4-5, and supine sagittal MR image of a 45-
year-old woman who has had back and leg pain for 2 years. Treatment had
included nonsteroidal anti-inflammatory drugs, physical therapy, and epidural
corticosteroid injections. Her pain limited her activities of daily living; she
could walk only 1 to 2 blocks before her pain became intolerable.

21A B

21
Adult spine self-scored self-assessment examination AAOS 2015

Question 21 of 100

A lateral transpsoas interbody fusion with percutaneous pedicle screws was


performed. When compared with those undergoing open laminectomy and
fusion, this patient can expect

1- a longer hospital stay.


2- a higher complication rate.
3- higher risk for postoperative thigh pain.
4- no relief of lower extremity symptoms.

PREFERRED RESPONSE: 3- higher risk for postoperative thigh pain.

Question 22 of 100

The above surgery was performed with recombinant human bone


morphogenetic protein-2 (rhBMP-2). She reports immediate relief of her leg
pain and is discharged home on postoperative day 2. The patient reports new-
onset radicular pain and weakness in her leg 1 year later. Figures 22a and 22b
are her postoperative CT myelogram images. What is the most likely cause of
her pain?

22
Adult spine self-scored self-assessment examination AAOS 2015

22A B

1- Excessive retraction of the traversing nerve root


2- Bony overgrowth from the use of BMP protein in the disk space
3- Osteomyelitis with compression of the nerve roots
4- Inadequate decompression after surgery

PREFERRED RESPONSE: 2- Bony overgrowth from the use of BMP protein in


the disk space

DISCUSSION

Transient thigh pain after lateral, transpsoas interbody fusion is common and
generally lasts for less than 3 months. Transpsoas interbody fusion with slip
reduction can result in indirect spinal decompression and often obviates the
need for a laminectomy for most patients. Complications with this approach
are comparable to those experienced with open surgery, but the hospital stay
is generally shorter.
Use of BMP in interbody devices has also become common. The images above
show heterotopic bone growth into the spinal canal causing nerve
compression. This complication is more commonly encountered after posterior
lumbar interbody fusions.

RECOMMENDED READINGS

Oliveira L, Marchi L, Coutinho E, Pimenta L. A radiographic assessment of the ability of the


extreme lateral interbody fusion procedure to indirectly decompress the neural elements.

23
Adult spine self-scored self-assessment examination AAOS 2015

Spine (Phila Pa 1976). 2010 Dec 15;35(26 Suppl):S331-7. doi:


10.1097/BRS.0b013e3182022db0. PubMed PMID: 21160397. View Abstract at PubMed
Cummock MD, Vanni S, Levi AD, Yu Y, Wang MY. An analysis of postoperative thigh symptoms
after minimally invasive transpsoas lumbar interbody fusion. J Neurosurg Spine. 2011
Jul;15(1):11-8. doi: 10.3171/2011.2.SPINE10374. Epub 2011 Apr 8. PubMed PMID:
21476801.View Abstract at PubMed
Rodgers WB, Gerber EJ, Patterson J. Intraoperative and early postoperative complications in
extreme lateral interbody fusion: an analysis of 600 cases. Spine (Phila Pa 1976). 2011 Jan
1;36(1):26-32. doi: 10.1097/BRS.0b013e3181e1040a. PubMed PMID: 21192221. View
Abstract at PubMed
Wong DA, Kumar A, Jatana S, Ghiselli G, Wong K. Neurologic impairment from ectopic bone
in the lumbar canal: a potential complication of off-label PLIF/TLIF use of bone morphogenetic
protein-2 (BMP-2). Spine J. 2008 Nov-Dec;8(6):1011-8. Epub 2007 Nov 26. PubMed PMID:
18037352.View Abstract at PubMed
Chen NF, Smith ZA, Stiner E, Armin S, Sheikh H, Khoo LT. Symptomatic ectopic bone
formation after off-label use of recombinant human bone morphogenetic protein-2 in
transforaminal lumbar interbody fusion. J Neurosurg Spine. 2010 Jan;12(1):40-6. doi:
10.3171/2009.4.SPINE0876. PubMed PMID: 20043763.View Abstract at PubMed
Rouben D, Casnellie M, Ferguson M. Long-term durability of minimal invasive posterior
transforaminal lumbar interbody fusion: a clinical and radiographic follow-up. J Spinal Disord
Tech. 2011 Jul;24(5):288-96. doi:10.1097/BSD.0b013e3181f9a60a. PubMed PMID:
20975594.View Abstract at PubMed

Question 23 of 100

Standard guidelines necessitate the use of intraoperative neurophysiological


monitoring for patients undergoing surgery for which condition?

1- Primary lumbar disk herniation


2- Adolescent idiopathic scoliosis
3- Cervical spondylotic myelopathy
4- No conditions; there are no official guidelines

PREFERRED RESPONSE: 4- No conditions; there are no official guidelines

DISCUSSION

There are currently no official guidelines on the appropriate use of


neuromonitoring in spine surgery. In general, use of neuromonitoring is at
surgeon discretion and often is based on the surgeon's perceived risk for
neurologic injury during surgery and medicolegal concerns. In most reports,

24
Adult spine self-scored self-assessment examination AAOS 2015

neuromonitoring is considered useful in cases of deformity correction, spinal


cord decompression, instrumentation placement, and revision surgery.
However, even for some of these cases, studies have shown limited benefits
of neuromonitoring and substantial associated costs.

RECOMMENDED READINGS

Lall RR, Lall RR, Hauptman JS, Munoz C, Cybulski GR, Koski T, Ganju A, Fessler RG, Smith
ZA. Intraoperative neurophysiological monitoring in spine surgery: indications, efficacy, and
role of the preoperative checklist. Neurosurg Focus. 2012 Nov;33(5):E10. doi:
10.3171/2012.9.FOCUS12235. Review. PubMed PMID: 23116090. View Abstract at PubMed
Peeling L, Hentschel S, Fox R, Hall H, Fourney DR. Intraoperative spinal cord and nerve root
monitoring: a survey of Canadian spine surgeons. Can J Surg. 2010 Oct;53(5):324-8. PubMed
PMID: 20858377. View Abstract at PubMed
Garces J, Berry JF, Valle-Giler EP, Sulaiman WA. Intraoperative neurophysiological monitoring
for minimally invasive 1- and 2-level transforaminal lumbar interbody fusion: does it improve
patient outcome? Ochsner J. 2014 Spring;14(1):57-61. PubMed PMID: 24688334. View
Abstract at PubMed

Question 24 of 100

A 56-year-old man has upper thoracic pain after undergoing stereotactic


radiosurgery for a blastic metastatic lesion in the vertebral body of T5. He has
normal alignment without collapse, but the tumor involves the entire vertebral
body. Which factor increases this patient's risk for a pathologic fracture?

1- Location of the lesion at T5


2- History of stereotactic radiation
3- Blastic nature of the tumor
4- Tumor involving all of the vertebral body

PREFERRED RESPONSE: 2- History of stereotactic radiation

DISCUSSION

Criteria for spinal instability have been outlined by the Spine Oncology Study
Group, which developed the Spinal Instability Neoplastic Score (SINS) criteria.
Factors associated with lower risk for instability/fracture are location outside
of a junctional level (the SINS criteria use C7-T2 as the junctional level),

25
Adult spine self-scored self-assessment examination AAOS 2015

blastic metastases, and no evidence of vertebral collapse (even with more


than 50% involvement). Radiation has been associated with risk for pathologic
fracture, and stereotactic radiation has been associated with risk for a spinal
fracture.

RECOMMENDED READINGS

Fisher CG, DiPaola CP, Ryken TC, Bilsky MH, Shaffrey CI, Berven SH, Harrop JS, Fehlings MG,
Boriani S, Chou D, Schmidt MH, Polly DW, Biagini R, Burch S, Dekutoski MB, Ganju A, Gerszten
PC, Gokaslan ZL, Groff MW, Liebsch NJ, Mendel E, Okuno SH, Patel S, Rhines LD, Rose PS,
Sciubba DM, Sundaresan N, Tomita K, Varga PP, Vialle LR, Vrionis FD, Yamada Y, Fourney
DR. A novel classification system for spinal instability in neoplastic disease: an evidence-based
approach and expert consensus from the Spine Oncology Study Group. Spine (Phila Pa 1976).
2010 Oct 15;35(22):E1221-9. doi: 10.1097/BRS.0b013e3181e16ae2. Review. PubMed
PMID:20562730. View Abstract at PubMed
Fourney DR, Frangou EM, Ryken TC, Dipaola CP, Shaffrey CI, Berven SH, Bilsky MH, Harrop
JS, Fehlings MG, Boriani S, Chou D, Schmidt MH, Polly DW, Biagini R, Burch S, Dekutoski MB,
Ganju A, Gerszten PC, Gokaslan ZL, Groff MW, Liebsch NJ, Mendel E, Okuno SH, Patel S,
Rhines LD, Rose PS, Sciubba DM, Sundaresan N, Tomita K, Varga PP, Vialle LR, Vrionis FD,
Yamada Y, Fisher CG. Spinal instability neoplastic score: an analysis of reliability and validity
from the spine oncology study group. J Clin Oncol. 2011 Aug 1;29(22):3072-7.
doi:10.1200/JCO.2010.34.3897. Epub 2011 Jun 27. PubMed PMID: 21709187. View Abstract
at PubMed
Rose PS, Laufer I, Boland PJ, Hanover A, Bilsky MH, Yamada J, Lis E. Risk of fracture after
single fraction image-guided intensity-modulated radiation therapy to spinal metastases. J
Clin Oncol. 2009 Oct 20;27(30):5075-9. doi: 10.1200/JCO.2008.19.3508. Epub 2009 Sep 8.
PubMed PMID: 19738130 View Abstract at PubMed

Question 25 of 100

A B

26
Adult spine self-scored self-assessment examination AAOS 2015

What is the appropriate first step when confirming the diagnosis of a


neurologically intact, 73-year-old man who has the images shown in Figures
25a through 25c?

1- Order a prostate-specific antigen (PSA) level


2- Perform a 2-level corpectomy and reconstruction with tissue sent for
pathologic analysis
3- Fine-needle aspiration
4- Bone scan

PREFERRED RESPONSE: 1- Order a prostate-specific antigen (PSA) level

DISCUSSION

The radiograph and MR images show an osteoblastic lesion in the T9 and T8


vertebral bodies. In an older man, this finding most likely reveals metastatic
prostate cancer. The first and least invasive diagnostic step is to order a PSA
level. Gleave and associates found in a retrospective review of patients with
prostate cancer that isolated levels of PSA lower than 10 to 20 micrograms
per liter are rarely associated with bone metastasis. Vis and associates
documented that 10-year prostate cancer survival in a screened population
was higher than 60%, and in an unscreened population it was 24%. In a
neurologically intact patient with no evidence of neural compression or
instability, surgery is not indicated. Fine-needle aspiration may be performed,
but the diagnostic yield in a blastic lesion is low. A bone scan may be indicated
to complete the metastatic workup, but it will not aid in the diagnosis of tissue
source.

27
Adult spine self-scored self-assessment examination AAOS 2015

RECOMMENDED READINGS

Gleave ME, Coupland D, Drachenberg D, Cohen L, Kwong S, Goldenberg SL, Sullivan LD.
Ability of serum prostate-specific antigen levels to predict normal bone scans in patients with
newly diagnosed prostate cancer. Urology. 1996 May;47(5):708-12. PubMed PMID: 8650870.
View Abstract at PubMed
Vis AN, Roemeling S, Reedijk AM, Otto SJ, Schröder FH. Overall survival in the intervention
arm of a randomized controlled screening trial for prostate cancer compared with a clinically
diagnosed cohort. Eur Urol. 2008 Jan;53(1):91-8. Epub 2007 Jun 12. PubMed PMID:
17583416. View Abstract at PubMed
Cronen GA, Emery SE. Benign and malignant lesions of the spine. In: Spivak JM, Connolly PJ,
eds. Orthopaedic Knowledge Update: Spine 3. Rosemont, IL: American Academy of
Orthopaedic Surgeons; 2006:351-366.

CLINICAL SITUATION FOR QUESTIONS 26 THROUGH 29

Figures 26a and 26b are the MR images with gadolinium enhancement of a
40-year-old man who arrives at the emergency department with a 4-day
history of fevers and severe back pain without radiation. He is normotensive
at presentation with a heart rate of 86 beats per minute. Upon examination
he is neurologically intact with normal sensory and motor function. He has a
history of alcohol and cocaine abuse. His white blood cell (WBC) count is
12000 (reference range [rr], 4500-11000 /µL) and his C-reactive protein
(CRP) level is 100 mg/L (rr, 0.08-3.1 mg/L)

Question 26 of 100

28
Adult spine self-scored self-assessment examination AAOS 2015

Based on this patient's history and examination, what is the best next step?

1- Order a confirmatory erythrocyte sedimentation rate (ESR)


2- Obtain blood cultures
3- Initiate empiric broad-spectrum antibiotics
4- Perform surgical decompression

PREFERRED RESPONSE: 2- Obtain blood cultures

Question 27 of 100

Blood cultures reveal methicillin-sensitive Staphylococcus aureus (MSSA). The


patient's examination remains unchanged. In addition to outpatient serial
laboratory studies and weekly observation for neurologic deterioration, which
of the following is the most appropriate for nonoperative treatment?

1- Interventional radiology placement of an epidural drain


2- An 8-week course of intravenous (IV) vancomycin
3- An 8-week course of IV nafcillin
4- An 8-week course of oral amoxicillin

PREFERRED RESPONSE: 3- An 8-week course of IV nafcillin

Question 28 of 100

Clinical characteristics of patients who are ideal for a nonsurgical treatment


trial (instead of surgery) include

1- absence of neurologic deficits and MSSA infection.


2- normal blood pressure and normal heart rate.
3- negative blood cultures, cocaine abuse
4- normal blood pressure and normal heart rate

PREFERRED RESPONSE: 1- absence of neurologic deficits and MSSA infection.

Question 29 of 100

29
Adult spine self-scored self-assessment examination AAOS 2015

What would be the advantage of surgery for the patient described in this
scenario?

1- Shorter length of antibiotic therapy


2- Reduced mortality risk
3- Reduced risk for neurologic deterioration
4- Reduced risk for chronic pain

PREFERRED RESPONSE: 3- Reduced risk for neurologic deterioration

DISCUSSION

The MR image of the lumbar spine postgadolinium contrast shows a ring-


enhancing fluid collection. Ring-enhancing lesions within the spinal canal on
postgadolinium MR images are indicative of epidural abscess. It is important
to obtain a culture-specific diagnosis to inform the choice of antibiotics and
educate patients regarding the likelihood of failure for standalone antibiotic
therapy. Infection with MSSA, age younger than 65 years, the absence of
neurologic deficit, and lumbar abscess location are all factors that point toward
a patient being a reasonable candidate for a trial of culture-specific IV
antibiotics. In this case, nafcillin is a suitable treatment for MSSA. The
antibiotic should be initiated and closely observed with serial labs (WBC, ESR,
CRP, repeat blood cultures) to ensure that the patient responds appropriately
to therapy and that neurologic deficits do not develop. In the setting of
epidural abscess, surgery is performed to evacuate the abscess and reverse
or prevent neurologic deterioration. In the current scenario in which sepsis is
not an issue, scant high-quality evidence shows that surgical intervention
influences risk for mortality or chronic pain following epidural abscess.

RECOMMENDED READINGS

Patel AR, Alton TB, Bransford RJ, Lee MJ, Bellabarba CB, Chapman JR. Spinal epidural
abscesses: risk factors, medical versus surgical management, a retrospective review of 128
cases. Spine J. 2014 Feb 1;14(2):326-30. doi: 10.1016/j.spinee.2013.10.046. Epub 2013
Nov 12. PubMed PMID: 24231778.View Abstract at PubMed
Adogwa O, Karikari IO, Carr KR, Krucoff M, Ajay D, Fatemi P, Perez EL, Cheng JS, Bagley CA,
Isaacs RE. Spontaneous spinal epidural abscess in patients 50 years of age and older: a 15-
year institutional perspective and review of the literature: clinical article. J Neurosurg Spine.
2014 Mar;20(3):344-9. doi: 10.3171/2013.11.SPINE13527. Epub 2013 Dec 20. Review.
PubMed PMID: 24359002.View Abstract at PubMed
Kim SD, Melikian R, Ju KL, Zurakowski D, Wood KB, Bono CM, Harris MB. Independent
predictors of failure of nonoperative management of spinal epidural abscesses. Spine J. 2014

30
Adult spine self-scored self-assessment examination AAOS 2015

Aug 1;14(8):1673-9. doi: 10.1016/j.spinee.2013.10.011. Epub 2013 Oct 30. PubMed PMID:
24373683.View Abstract at PubMed
Schoenfeld AJ. Spine infections. In: Cannada L, ed. Orthopaedic Knowledge Update 11.
Rosemont, IL: American Academy of Orthopaedic Surgeons, 2014: 737-747.

Question 30 of 100

A B

C
Figures 30a through 30c are the radiograph and MR images of a 54-year-old woman
who has severe leg pain with walking. Her treatment has included 12 weeks of

31
Adult spine self-scored self-assessment examination AAOS 2015

physical therapy, anti-inflammatory medications, and narcotic pain relievers, and she
is interested in surgery. Minimally invasive transforaminal lumbar interbody fusion
(MIS TLIF) is recommended. When compared with open TLIF, MIS TLIF is associated
with

1- shorter hospital stay.


2- lower rates of fusion.
3- higher complication rates.
4- worse long-term outcomes.

PREFERRED RESPONSE: 1- shorter hospital stay.

DISCUSSION

MIS TLIF involves a steep learning curve but is associated with similar long-
term outcomes as open TLIF, arguably comparable or possibly lower
complication rates, and equivalent fusion rates. The major distinguishing
feature comparing open to minimally invasive surgery for this and other spinal
diagnoses has been shorter hospital stays.

RECOMMENDED READINGS

Peng CW, Yue WM, Poh SY, Yeo W, Tan SB. Clinical and radiological outcomes of minimally
invasive versus open transforaminal lumbar interbody fusion. Spine (Phila Pa 1976). 2009 Jun
1;34(13):1385-9. doi: 10.1097/BRS.0b013e3181a4e3be. PubMed PMID: 19478658. View
Abstract at PubMed
Lee KH, Yue WM, Yeo W, Soeharno H, Tan SB. Clinical and radiological outcomes of open
versus minimally invasive transforaminal lumbar interbody fusion. Eur Spine J. 2012
Nov;21(11):2265-70. doi: 10.1007/s00586-012-2281-4. Epub 2012 Mar 28. PubMed PMID:
22453894. View Abstract at PubMed
Lau D, Lee JG, Han SJ, Lu DC, Chou D. Complications and perioperative factors associated
with learning the technique of minimally invasive transforaminal lumbar interbody fusion
(TLIF). J Clin Neurosci. 2011 May;18(5):624-7. doi: 10.1016/j.jocn.2010.09.004. Epub 2011
Feb 23. PubMed PMID: 21349719. View Abstract at PubMed

Question 31 of 100

An orthopaedic surgeon is counseling a patient regarding risk for complications


following lumbar fusion via a direct lateral approach. Surgery at which level is
most likely to injure the lumbosacral plexus?

32
Adult spine self-scored self-assessment examination AAOS 2015

1- L1-2
2- L2-3
3- L3-4
4- L4-5

PREFERRED RESPONSE: 4- L4-5

DISCUSSION

During the direct lateral approach, interbody fusion devices are inserted
through a lateral window in the psoas muscle. To accomplish this, dilators and
retractors are positioned at the posterior half of the disk space, and it must
be noted that the lumbosacral plexus lies within the psoas muscle between
the transverse process and vertebral body and departs distally at the medial
edge of the psoas. Consequently, lateral interbody fusion poses risk for injury
to the lumbosacral plexus. A cadaveric study demonstrated that the
lumbosacral plexus progressively migrates from dorsal to ventral in the lumbar
spine. Therefore, the plexus is most likely to be injured during an L4-L5 fusion
because at this level the lumbosacral plexus is closest to the location at which
dilators and retractors are placed.
A 2013 retrospective study by Le and associates followed 71 patients who
underwent minimally invasive fusion via a lateral interbody approach. In this
study, 54.9% (39/71) had immediate postsurgical ipsilateral iliopsoas or
quadriceps weakness. Of these patients, the majority had resolution by 3
months (92.3%), and all had complete resolution by 2 years.

RECOMMENDED READINGS

Le TV, Burkett CJ, Deukmedjian AR, Uribe JS. Postoperative lumbar plexus injury after lumbar
retroperitoneal transpsoas minimally invasive lateral interbody fusion. Spine (Phila Pa 1976).
2013 Jan 1;38(1):E13-20. doi: 10.1097/BRS.0b013e318278417c. PubMed PMID: 23073358.
View Abstract at PubMed
Benglis DM, Vanni S, Levi AD. An anatomical study of the lumbosacral plexus as related to
the minimally invasive transpsoas approach to the lumbar spine. J Neurosurg Spine. 2009
Feb;10(2):139-44. doi: 10.3171/2008.10.SPI08479. PubMed PMID: 19278328. View Abstract
at PubMed
Knight RQ, Schwaegler P, Hanscom D, Roh J. Direct lateral lumbar interbody fusion for
degenerative conditions: early complication profile. J Spinal Disord Tech. 2009 Feb;22(1):34-
7. doi: 10.1097/BSD.0b013e3181679b8a. PubMed PMID: 19190432.View Abstract at PubMed

33
Adult spine self-scored self-assessment examination AAOS 2015

Question 32 of 100

Figure 32 shows the T2-weighted MR image through the L4-5 level of a 60-
year-old man who has new-onset acute right lower-extremity pain and
numbness and weakness in his right quadriceps muscle. The arrow in Figure
32 is pointing to which structure?

1- Lumbar synovial cyst


2- Dorsal root ganglion
3- Herniated nucleus pulposus
4- Ligamentum flavum

PREFERRED RESPONSE: 3- Herniated nucleus pulposus

DISCUSSION

The arrow is pointing to a structure of medium signal intensity that is


equivalent to the nucleus pulposus on T2-weighted sequencing. This
represents a foraminal disk herniation. A lumbar synovial cyst would display
high-signal intensity on T2-weighted sequencing. Lumbar synovial cysts arise
from the facet joints as a result of facet joint degeneration and may be a
source of nerve root compression. The dorsal root ganglion is a collection of
sensory nerve cell bodies and can be seen just dorsal and lateral to the disk
herniation in Figure 32. The ligamentum flavum is located on the ventral

34
Adult spine self-scored self-assessment examination AAOS 2015

surface of the laminae and attaches between the laminae of adjacent


vertebrae.

RECOMMENDED READINGS

Patel NM, Jenis LG. Inflammatory arthritis of the spine. In: Spivak JM, Connolly PJ, eds.
Orthopaedic Knowledge Update: Spine 3. Rosemont, IL: American Academy of Orthopaedic
Surgeons; 2006:339-349.
Carrino JA, Morrison WB. Musculoskeletal imaging. In: Vaccaro AR, ed. Orthopaedic
Knowledge Update 8. Rosemont, IL: American Academy of Orthopaedic Surgeons; 2005:119-
136.

Question 33 of 100

One year after undergoing anterior cervical decompression and fusion, what
percentage of patients still have dysphagia?

1- 10% to 15%
2- 30% to 35%
3- 50% to 55%
4- 70% to 75%

PREFERRED RESPONSE: 1- 10% to 15%

DISCUSSION

Dysphagia after anterior cervical diskectomy and fusion is a common, usually


transient finding after anterior cervical approaches to the spine. While it has
been reported to occur in up to 70% of patients 2 weeks following surgery, in
most cases the symptoms quickly resolve. There is, however, a small subset
of patients for whom symptoms of dysphagia will persist. Lee and associates
prospectively studied the rate of dysphagia after anterior cervical diskectomy
and fusion, reporting a 15% rate of dysphagia at 12 months, and 12% at 24
months. Phillips and associates analyzed the 2-year data from the PCM FDE
clinical trial and found a 12.1% incidence of dysphagia in the ACDF arm.

RECOMMENDED READINGS

35
Adult spine self-scored self-assessment examination AAOS 2015

Lee MJ, Bazaz R, Furey CG, Yoo J. Risk factors for dysphagia after anterior cervical spine
surgery: a two-year prospective cohort study. Spine J. 2007 Mar-Apr;7(2):141-7. Epub 2007
Jan 22. PubMed PMID: 17321961. View Abstract at PubMed
Smith-Hammond CA, New KC, Pietrobon R, Curtis DJ, Scharver CH, Turner DA. Prospective
analysis of incidence and risk factors of dysphagia in spine surgery patients: comparison of
anterior cervical, posterior cervical, and lumbar procedures. Spine (Phila Pa 1976). 2004 Jul
1;29(13):1441-6. PubMed PMID: 15223936. View Abstract at PubMed
Edwards CC 2nd, Karpitskaya Y, Cha C, Heller JG, Lauryssen C, Yoon ST, Riew KD. Accurate
identification of adverse outcomes after cervical spine surgery. J Bone Joint Surg Am. 2004
Feb;86-A(2):251-6. PubMed PMID: 14960668. View Abstract at PubMed
Phillips FM, Lee JY, Geisler FH, Cappuccino A, Chaput CD, DeVine JG, Reah C, Gilder KM,
Howell KM, McAfee PC. A prospective, randomized, controlled clinical investigation comparing
PCM cervical disc arthroplasty with anterior cervical discectomy and fusion. 2-year results
from the US FDA IDE clinical trial. Spine (Phila Pa 1976). 2013 Jul 1;38(15):E907-18. doi:
10.1097/BRS.0b013e318296232f.
Rihn JA, Kane J, Albert TJ, Vaccaro AR, Hilibrand AS. What is the incidence and severity of
dysphagia after anterior cervical surgery? Clin Orthop Relat Res. 2011 Mar;469(3):658-65.
PMID: 21140251.View Abstract at PubMed

Question 34 of 100

What is the most appropriate initial diagnostic imaging study for a patient with
presumed diskogenic low-back pain?

1- MRI
2- Discography
3- CT discography
4- Radiography

PREFERRED RESPONSE: 4- Radiography

DISCUSSION

Radiography is the best initial study. Vacuum phenomenon may be found


within the disk space. Other possible sources for back pain such as
osteoporotic collapse, osteolytic collapse, and deformity also can be
evaluated. The other tests may be beneficial and are more appropriate as later
imaging options.

RECOMMENDED READINGS

36
Adult spine self-scored self-assessment examination AAOS 2015

Yu WD, Williams SL. Spinal imaging: Radiographs, computed tomography, and magnetic
resonance imaging. In: Spivak JM, Connolly PJ, eds. Orthopaedic Knowledge Update: Spine
3. Rosemont, IL: American Academy of Orthopaedic Surgeons; 2006:57-67.
Bess RS, Brodke DS. Degenerative disease of the lumbar spine. In: Fischgrund JS, ed.
Orthopaedic Knowledge Update 9. Rosemont, IL: American Academy of Orthopaedic
Surgeons; 2008:551-563.

Question 35 of 100

Figure 35 is the sagittal MR image of a 56-year-old woman who has a 3-year


history of severe back pain. Her pain is worse with flexion at the lumbosacral
junction and is relieved with extension. She denies any pain in her lower
extremities and has no symptoms of neurogenic claudication. Which mediators
play roles in the pathogenesis of this condition?

1- Transforming growth factor-beta (TGF-ß), bone morphogenetic protein-2


(BMP-2), latent membrane protein 1
2- Tissue inhibitor of matrix metallo-proteinase-1 (MMP-1), growth and
development factor-5, noggin
3- Gremlin, MMP, biglycan
4- Tumor necrosis factor-alpha (TNF-a), Interleukin-1 (IL-1), MMP

37
Adult spine self-scored self-assessment examination AAOS 2015

PREFERRED RESPONSE: 4- Tumor necrosis factor-alpha (TNF-a), Interleukin-


1 (IL-1), MMP

DISCUSSION

The patient has degenerative disk disease with diskogenic back pain. Several
studies in both humans and animals have implicated TNF-a, IL-1, and MMP in
extracellular matrix degeneration and disk degradation. TGF-ß, BMP-2, latent
membrane protein 1, and growth and development factor-5 are all postulated
to play anabolic roles in the intervertebral disk. Biglycan is a small leucine-
rich proteoglycan that regulates extracellular matrix assembly within the disk.
Noggin and gremlin are biochemical factors not involved in disk degradation.

RECOMMENDED READINGS

Kim HT, Yoon ST, Jarrett C. Articular cartilage and intervertebral disk. In: Fischgrund JS, ed.
Orthopaedic Knowledge Update 9. Rosemont, IL: American Academy of Orthopaedic
Surgeons; 2008:23-33.
Hoyland JA, Le Maitre C, Freemont AJ. Investigation of the role of IL-1 and TNF in matrix
degradation in the intervertebral disc. Rheumatology (Oxford). 2008 Jun;47(6):809-14. doi:
10.1093/rheumatology/ken056. Epub 2008 Apr 8. PubMed PMID: 18397957. View Abstract
at PubMed
Gruber HE, Ingram JA, Hanley EN Jr. Immunolocalization of MMP-19 in the human
intervertebral disc: implications for disc aging and degeneration. Biotech Histochem. 2005
May-Aug;80(3-4):157-62. PubMed PMID: 16298901. View Abstract at PubMed

Question 36 of 100

A 45-year-old woman with a history of rheumatoid arthritis has C1-C2


instability with neurologic deterioration. Her posterior atlanto-dens interval is
10 mm. Which fixation technique will be the most biomechanically sound to
facilitate fusion across the atlanto-axial junction?

1- Gallie fusion
2- Use of C1-C2 transarticular screws
3- Brooks fusion
4- Onlay grafting with a halo vest

PREFERRED RESPONSE: 2- Use of C1-C2 transarticular screws

38
Adult spine self-scored self-assessment examination AAOS 2015

DISCUSSION

C1-C2 transarticular screw fixation is 10-fold stiffer than wiring constructs,


particularly in rotation; eliminates the need for postsurgical halo use; and is
associated with reported fusion rates to a maximum of 100% for bilateral
screws and 95% for unilateral fixation. All of the other fusion techniques
mentioned are associated with a pseudarthrosis rate of at least 30%.

RECOMMENDED READINGS

Stock GH, Vaccaro AR, Brown AK, Anderson PA. Contemporary posterior occipital fixation. J
Bone Joint Surg Am. 2006 Jul;88(7):1642-9. PubMed PMID: 16841419. View Abstract at
PubMed
Sim HB, Lee JW, Park JT, Mindea SA, Lim J, Park J. Biomechanical evaluations of various c1-
c2 posterior fixation techniques. Spine (Phila Pa 1976). 2011 Mar 15;36(6):E401-7. doi:
10.1097/BRS.0b013e31820611ba. PubMed PMID: 21372651. View Abstract at PubMed

CLINICAL SITUATION FOR QUESTIONS 37 THROUGH 43

Figure 37 is the lateral radiograph of a 71-year-old woman who has pain with
ambulation that improves when she sits down. She had similar symptoms 2 years
earlier when she underwent an L3-L5 posterior spinal fusion. Upon examination she
has good range of hip and knee motion, 5/5 motor function, and normal sensation of
her lower extremities. She has negative bilateral straight-leg raise findings and her
knees slightly flex to stand upright

37

39
Adult spine self-scored self-assessment examination AAOS 2015

Question 37 of 100

What is the most likely diagnosis considering this patient's history and
examination?

1- Metastatic disease of the spine


2- Neurogenic claudication
3- Degenerative joint disease of the hips
4- Degenerative scoliosis

PREFERRED RESPONSE: 2- Neurogenic claudication

Question 38 of 100

In addition to MR imaging, what is the most appropriate additional


radiographic study at this time?

1- Mechanical axis of the lower extremities


2- Long-standing radiograph of the entire spine
3- Nuclear medicine: triple-phase bone scan
4- Hyperextension radiograph of the lumbar spine

PREFERRED RESPONSE: 2- Long-standing radiograph of the entire spine

Question 39 of 100

In addition to her planned primary procedure, how can the surgeon best
improve this patient's lumbar lordosis?

1- Placement of bilateral iliac screws


2- Use of an interbody strut at L5-S1
3- Extension of instrumentation to T11
4- Extension lumbosacral orthosis

PREFERRED RESPONSE: 2- Use of an interbody strut at L5-S1

40
Adult spine self-scored self-assessment examination AAOS 2015

Question 40 of 100

The patient is treated by another physician and undergoes the procedure


shown in Figures 40a and 40b. She currently cannot stand upright. Spinopelvic
measurements reveal her pelvic incidence to be 85 degrees and pelvic tilt to
be 40 degrees. Her lordosis from T12-S1 is 35 degrees. Which additional
surgical procedure should be considered?

40A B

1- Single-level pedicle subtraction osteotomy


2- Single-level Smith-Petersen osteotomy
3- Instrumentation removal
4- Percutaneous fusion of the sacroiliac joints

PREFERRED RESPONSE: 1- Single-level pedicle subtraction osteotomy

Question 41 of 100

The patient's postsurgical radiographs reveal a sagittal vertical axis of +8 cm.


In addition to the usual issues encountered during the early postsurgical
period, what should the surgeon be most concerned about?

41
Adult spine self-scored self-assessment examination AAOS 2015

1- Instrumentation-related pain
2- Broken rod
3- Sacroiliac dissociation
4- Proximal junctional failure

PREFERRED RESPONSE: 4- Proximal junctional failure

Question 42 of 100

Bearing in mind her current condition as shown in Figures 40a and 40b, the
surgeon should inform the patient and family that if she is to stand erect, she

1- needs further spinal extensor strengthening.


2- will need to wear a spinal brace indefinitely.
3- will need another spinal surgery.
4- will need a spinal cord stimulator.

PREFERRED RESPONSE: 3- will need another spinal surgery.

Question 43 of 100

If additional posterior spinal surgery is performed to allow the patient to stand


erect, the surgeon will need to advise the family about the risks of the
procedure. In addition to risk for neurologic injury, what is the most likely
complication?

1- Substantial blood loss


2- Cerebral vascular accident
3- Degeneration of sacroiliac joints
4- Cervical spine stenosis

PREFERRED RESPONSE: 1- Substantial blood loss

DISCUSSION

This case starts as standard degenerative spondylolisthesis but develops into


a sagittal imbalance problem necessitating a major spinal procedure including

42
Adult spine self-scored self-assessment examination AAOS 2015

a pedicle subtraction osteotomy and extended posterior spinal


instrumentation. Initially the patient had an L3-L5 posterior spinal fusion with
a laminectomy and interbody fusion for an L4-L5 stenosis and degenerative
spondylolisthesis. Two years after this procedure she is having difficult
ambulation that improves with sitting. These symptoms are typical for
neurogenic claudication associated with spinal stenosis. Adjacent segment
degeneration leading to spinal stenosis is a common late complication
associated with lumbar instrumented fusions, particularly in older patients.
Figure 40b shows the adjacent segment degeneration at L5-S1 with the
development of a degenerative spondylolisthesis. Degenerative
spondylolisthesis is a radiographic sign of substantial degeneration of a disk
space and is often associated with spinal stenosis. The primary procedure
typically planned to address a degenerative spondylolisthesis is a
laminectomy. An adjacent-level degenerative spondylolisthesis is typically
instrumented and fused to the previous fusion construct.
The degeneration of the adjacent disk is also associated with a loss of disk
height. A concern in this patient is the development of kyphosis at the L5-S1
disk space. Kyphosis at the lumbosacral junction can lead to sagittal imbalance
issues. Spinal surgeons need to be aware of spinopelvic measurements to help
prevent sagittal imbalance and proximal junctional failures. Improving the
lumbar lordosis in this patient would necessitate restoration of the disk height
at L5-S1. This can be accomplished with an interbody strut device placed
either anteriorly or posteriorly via a posterolateral interbody approach at the
time of the laminectomy.
This patient has undergone a posterior extension of her previous fusion
without the addition of an interbody strut device. She now has further sagittal
imbalance issues and an inability to stand erect without flexing her knees. This
is a typical compensation posture that patients with significant sagittal
imbalance acquire when trying to stand upright. Patients with sagittal
imbalance are at increased risk for proximal junctional kyphosis or a more
acute complication of proximal junctional failure.
The sagittal vertical axis is a plumb line dropped from C7 and should fall
behind the hip joints and within 4 to 5 cm of the posterior corner of S1 (an
easily identifiable radiographic marker). Pelvic incidence is a constant that is
unique to each patient's spinopelvic anatomy. Pelvic incidence typically is
within 10 degrees of the lumbar lordosis in an upright adult. Pelvic tilt (PT),
on the other hand, can vary based on a patient's stance. PT is an indicator of
the amount of compensation a patient has developed by retroverting their
pelvis to stand upright. In an upright patient who is not compensating for loss
of spinal sagittal alignment, the PT should be less than 20 degrees. In this
case, the patient has a high PT and a significant lumbar lordosis/pelvic
incidence mismatch with a significant positive sagittal imbalance (sagittal
vertical axis of +8 cm). These are all indicators that a major spinal alignment
procedure will be required to rebalance the spine. The use of posterior

43
Adult spine self-scored self-assessment examination AAOS 2015

osteotomies, such as a pedicle subtraction osteotomy, will be required if spinal


realignment is planned. Pedicle subtraction osteotomies are considered 3-
column osteotomies that remove the entire lamina, the facets, the pedicles
(the posterior column), the underlying posterior vertebral wall and posterior
vertebral body (middle column), and the underlying anterior vertebral body
(anterior column) in a wedge fashion. Three-column osteotomies are
associated with increased risk for neurologic injury and substantial blood loss.
A Smith-Petersen osteotomy is a single-column posterior osteotomy that can
provide a lesser amount of sagittal plane correction than pedicle subtraction
osteotomy. A single-level Smith-Petersen osteotomy likely will not provide
enough correction in this case.

RECOMMENDED READINGS

Lafage V, Schwab F, Vira S, Patel A, Ungar B, Farcy JP. Spino-pelvic parameters after surgery
can be predicted: a preliminary formula and validation of standing alignment. Spine (Phila Pa
1976). 2011 Jun;36(13):1037-45. doi: 10.1097/BRS.0b013e3181eb9469. PubMed PMID:
21217459. View Abstract at PubMed
Schwab F, Patel A, Ungar B, Farcy JP, Lafage V. Adult spinal deformity-postoperative standing
imbalance: how much can you tolerate? An overview of key parameters in assessing
alignment and planning corrective surgery. Spine (Phila Pa 1976). 2010 Dec 1;35(25):2224-
31. doi: 10.1097/BRS.0b013e3181ee6bd4. Review. PubMed PMID: 21102297. View Abstract
at PubMed

Question 44 of 100

A 78-year-old athletic woman has a history of severe back pain without


antecedent trauma. She was in the emergency department 2 days ago with a
T12 compression fracture. A dual x-ray absorptiometry (DEXA) scan
performed earlier this year revealed a T-score of -2.8. Her condition may be
attributable to

1- translocation of chromosomes X and 18.


2- variant of the EXT1 gene.
3- polymorphism of p53.
4- polymorphism of the COL1A1 gene.

PREFERRED RESPONSE: 4- polymorphism of the COL1A1 gene.

44
Adult spine self-scored self-assessment examination AAOS 2015

DISCUSSION

The diagnosis is severe osteoporosis because this patient's T-score is lower


than -2.5 on DEXA scan and her fragility fracture involves the T12 vertebra.
Osteoporosis may be associated with polymorphisms of the COL1A1 gene.
EXT1 is associated with multiple hereditary exostoses, and translocation X:18
is associated with synovial sarcoma. The gene p53 is associated with Li-
Fraumeni syndrome and osteosarcoma.

RECOMMENDED READINGS

Masoodi TA, Alsaif MA, Al Shammari SA, Alhamdan AA. Evaluation and identification of
damaged single nucleotide polymorphisms in COL1A1 gene involved in osteoporosis. Arch
Med Sci. 2013 Oct 31;9(5):899-905. doi: 10.5114/aoms.2012.28598. Epub 2012 May 13.
PubMed PMID: 24273577. ? View Abstract at PubMed
Kurt-Sirin O, Yilmaz-Aydogan H, Uyar M, Seyhan MF, Isbir T, Can A. Combined effects of
collagen type I alpha1 (COL1A1) Sp1 polymorphism and osteoporosis risk factors on bone
mineral density in Turkish postmenopausal women. Gene. 2014 May 1;540(2):226-31. doi:
10.1016/j.gene.2014.02.028. Epub 2014 Feb 22. PubMed PMID: 24566004. ? View Abstract
at PubMed

Question 45 of 100

A B

Figures 45a and 45b are the CT and MR spine images of an 82-year-old man
who has a history of ankylosing spondylitis falls onto his back. He has no

45
Adult spine self-scored self-assessment examination AAOS 2015

neurologic deficits upon examination in the emergency department. What is


the most appropriate next step?

1- Obtain upright radiographs


2- Thoracolumbar orthosis
3- Posterior stabilization and fusion
4- Laminectomy

PREFERRED RESPONSE: 3- Posterior stabilization and fusion

DISCUSSION

Spinal fractures in patients with ankylosing spondylitis are unstable and


generally necessitate surgical intervention. In a patient with a spinal fracture
in the setting of ankylosing spondylitis, posterior instrumented fusion is an
appropriate surgical procedure. Treatment with a thoracolumbar orthosis is
not an option for patients with extension distraction injuries in the setting of
an ankylosed spine because of risk for displacement. Similarly, simply
checking upright radiographs is generally not advocated. Laminectomy alone
is inappropriate for this patient because there is no cord compression and
neurologic symptoms are absent. Stabilization is the treatment goal.

RECOMMENDED READINGS

Braun J, Sieper J. Ankylosing spondylitis. Lancet. 2007 Apr 21;369(9570):1379-90. Review.


PubMed PMID: 17448825.View Abstract at PubMed
Caron T, Bransford R, Nguyen Q, Agel J, Chapman J, Bellabarba C. Spine fractures in patients
with ankylosing spinal disorders. Spine (Phila Pa 1976). 2010 May 15;35(11):E458-64. doi:
10.1097/BRS.0b013e3181cc764f. PubMed PMID: 20421858. View Abstract at PubMed
Finkelstein JA, Chapman JR, Mirza S. Occult vertebral fractures in ankylosing spondylitis.
Spinal Cord. 1999 Jun;37(6):444-7. PubMed PMID: 10432265. View Abstract at PubMed
Hitchon PW, From AM, Brenton MD, Glaser JA, Torner JC. Fractures of the thoracolumbar
spine complicating ankylosing spondylitis. J Neurosurg. 2002 Sep;97(2 Suppl):218-22.
PubMed PMID: 12296682. View Abstract at PubMed

Question 46 of 100

46
Adult spine self-scored self-assessment examination AAOS 2015

A 17-year-old girl is involved in a motor vehicle collision and sustains the


injury shown in Figures 46a through 46c. She is neurologically intact in her
bilateral lower extremities. Definitive treatment should consist of

A B C

1- anterior reduction, stabilization, and fusion at L1-2.


2- anterior reduction, stabilization, and fusion at T12-L3.
3- posterior reduction, stabilization, and fusion at L1-2.
4- posterior reduction, stabilization, and fusion at T12-L4.

PREFERRED RESPONSE: 4- posterior reduction, stabilization, and fusion at


T12-L4.

DISCUSSION

The figures reveal a fracture-dislocation at L1-2. Proper treatment consists of


posterior reduction, stabilization, and fusion 2 levels above and below the level
of injury. Short-segment stabilization schemes do not stabilize the injury
properly, and longer-segment constructs are not necessary. Anterior
treatment is not indicated in fracture-dislocations.

RECOMMENDED READINGS

Mikles MR, Stchur RP, Graziano GP. Posterior instrumentation for thoracolumbar fractures. J
Am Acad Orthop Surg. 2004 Nov-Dec;12(6):424-35. Review. PubMed PMID: 15615508. View
Abstract at PubMed

47
Adult spine self-scored self-assessment examination AAOS 2015

Bono CM, Rinaldi MD. Thoracolumbar trauma. In: Spivak JM, Connolly PJ, eds. Orthopaedic
Knowledge Update: Spine 3. Rosemont, IL: American Academy of Orthopaedic Surgeons;
2006:201-216.

Question 47 of 100

What is the advantage of percutaneous pedicle screw fixation over open


instrumentation and fusion for a thoracolumbar burst fracture without
neurologic deficit?

1- Better clinical outcomes


2- Less blood loss
3- Less pseudarthrosis risk
4- Lower revision surgery rate

PREFERRED RESPONSE: 2- Less blood loss

DISCUSSION

A prospective randomized study on short-segment treatment of burst


fractures with and without fusion demonstrated similar outcomes at 5 years
with lower blood loss in the nonfusion group. There is by definition no fusion
performed with percutaneous stabilization, so patients often develop hardware
failure. Some surgeons routinely remove instrumentation following
percutaneous stabilization, thus revision surgery is common. Clinical
outcomes are not improved compared to open methods.

RECOMMENDED READINGS

Koreckij T, Park DK, Fischgrund J. Minimally invasive spine surgery in the treatment of
thoracolumbar and lumbar spine trauma. Neurosurg Focus. 2014;37(1):E11. doi:
10.3171/2014.5.FOCUS1494. Review. PubMed PMID: 24981899. View Abstract at PubMed
Jindal N, Sankhala SS, Bachhal V. The role of fusion in the management of burst fractures of
the thoracolumbar spine treated by short segment pedicle screw fixation: a prospective
randomised trial. J Bone Joint Surg Br. 2012 Aug;94(8):1101-6. doi: 10.1302/0301-
620X.94B8.28311. PubMed PMID: 22844053. View Abstract at PubMed
Dai LY, Jiang LS, Jiang SD. Posterior short-segment fixation with or without fusion for
thoracolumbar burst fractures. a five to seven-year prospective randomized study. J Bone

48
Adult spine self-scored self-assessment examination AAOS 2015

Joint Surg Am. 2009 May;91(5):1033-41. doi: 10.2106/JBJS.H.00510. PubMed PMID:


19411450. View Abstract at PubMed

RESPONSES FOR QUESTIONS 48 THROUGH 50

1- Fracture displacement or subluxation leading to neurologic injury


2- Symptomatic epidural hematoma
3- Sepsis and death
4- Permanent nerve root injury and neurologic deficit
5- Loss of bowel or bladder control
6- Spinal cord injury
Select the complication listed above that most commonly is associated
with the appropriate clinical scenario described below.

Question 48 of 100

A 25-year-old man with a large central disk herniation at L4-5 with normal
motor strength, perineal numbness, and difficulty urinating for 36 hours

1- Fracture displacement or subluxation leading to neurologic injury


2- Symptomatic epidural hematoma
3- Sepsis and death
4- Permanent nerve root injury and neurologic deficit
5- Loss of bowel or bladder control
6- Spinal cord injury

PREFERRED RESPONSE: 5- Loss of bowel or bladder control

Question 49 of 100

A 65-year-old man with ankylosing spondylitis has a fall from a standing


height. He previously had minimal range of motion in his cervical spine, but
now notices he is better able to extend his head. He is seen in the emergency
department and released with a soft collar for use as needed.

1- Fracture displacement or subluxation leading to neurologic injury


2- Symptomatic epidural hematoma

49
Adult spine self-scored self-assessment examination AAOS 2015

3- Sepsis and death


4- Permanent nerve root injury and neurologic deficit
5- Loss of bowel or bladder control
6- Spinal cord injury

PREFERRED RESPONSE: 1- Fracture displacement or subluxation leading to


neurologic injury

Question 50 of 100

A 74-year-old man with ankylosing spondylitis falls off a step stool and now
has a minimally displaced T10-T11 extension-type fracture. He is initially
treated with percutaneous pedicle screw fixation from T8-L1 and has good
pain relief. The next day, however, he experiences increased back pain and
loss of strength in his lower extremities.

1- Fracture displacement or subluxation leading to neurologic injury


2- Symptomatic epidural hematoma
3- Sepsis and death
4- Permanent nerve root injury and neurologic deficit
5- Loss of bowel or bladder control
6- Spinal cord injury

PREFERRED RESPONSE: 2- Symptomatic epidural hematoma

DISCUSSION

Cauda equina syndrome, typically the result of severe central canal stenosis
in the lower lumbar region, often is caused by a large central disk herniation.
Symptoms include severe back or leg pain, perineal numbness, possible motor
weakness, and initial urinary retention followed by an overflow incontinence.
When bowel or bladder deficits are present, this is considered a surgical
emergency because successful recovery is most likely if decompression occurs
within the first 48 hours.
Ankylosing spondylitis can lead to progressive autofusion of the vertebrae and
significant limitation in motion. Any sudden improvement in motion should be
considered a fracture until proven otherwise. These fractures are commonly
missed when using plain radiographs or even CT scan because minimal or no
displacement often is noted. MR imaging can be useful to identify edema at

50
Adult spine self-scored self-assessment examination AAOS 2015

the fracture site. These fractures are typically very unstable and necessitate
surgery to avoid displacement and potential neurologic injury.
Some fractures associated with ankylosing spondylitis can be effectively
treated with percutaneous pedicle screw fixation. However, because of the
highly vascular nature of some of these fractures, they pose risk for an
epidural hematoma, potential neurologic deficit, and emergent
decompression.

RECOMMENDED READINGS

Patel AR, Alton TB, Bransford RJ, Lee MJ, Bellabarba CB, Chapman JR. Spinal epidural
abscesses: risk factors, medical versus surgical management, a retrospective review of 128
cases. Spine J. 2014 Feb 1;14(2):326-30. doi: 10.1016/j.spinee.2013.10.046. Epub 2013
Nov 12. PubMed PMID: 24231778. View Abstract at PubMed
Mathews M, Bolesta MJ. Treatment of spinal fractures in ankylosing spondylitis. Orthopedics.
2013 Sep;36(9):e1203-8. doi: 10.3928/01477447-20130821-25. PubMed PMID: 24025014.
View Abstract at PubMed
McCarthy MJ, Aylott CE, Grevitt MP, Hegarty J. Cauda equina syndrome: factors affecting
long-term functional and sphincteric outcome. Spine (Phila Pa 1976). 2007 Jan 15;32(2):207-
16. PubMed PMID: 17224816. View Abstract at PubMed

Question 51 of 100

What is the chief mechanism of action of parathyroid hormone (PTH) in the


treatment of patients with osteoporosis?

1- Reduces resorptive activity of osteoclasts


2- Inhibits receptor activator of nuclear factor kappa-B ligand
3- Stimulates osteoblastic bone formation
4- Acts as a selective estrogen modulator

PREFERRED RESPONSE: 3- Stimulates osteoblastic bone formation

DISCUSSION

Recombinant human PTH benefits patients with osteoporosis by stimulating


osteoblastic bone formation and reducing osteoblastic apoptosis. Treatment
reduces vertebral fractures by 65%. PTH analogs act similarly and reduce
vertebral fractures by 47%. Bisphosphonates reduce the resorptive activity of

51
Adult spine self-scored self-assessment examination AAOS 2015

osteoclasts and cause a dissociation of bone formation and resorption that


favors bone formation and reduce vertebral fractures by 50% to 70%.
Selective estrogen receptor modulators inhibit bone resorption and reduce
vertebral fractures by 35%. Humanized monoclonal antibodies inhibit
osteoclast formation and reduce vertebral fractures by 68%.

RECOMMENDED READINGS

Castro-Lionard K, Dargent-Molina P, Fermanian C, Gonthier R, Cassou B. Use of calcium


supplements, vitamin D supplements and specific osteoporosis drugs among French women
aged 75-85 years: patterns of use and associated factors. Drugs Aging. 2013
Dec;30(12):1029-38. doi: 10.1007/s40266-013-0121-9. PubMed PMID: 24114665. View
Abstract at PubMed
Papapoulos S, Makras P. Selection of antiresorptive or anabolic treatments for
postmenopausal osteoporosis. Nat Clin Pract Endocrinol Metab. 2008 Sep;4(9):514-23. doi:
10.1038/ncpendmet0941. Review. PubMed PMID: 18714329. View Abstract at PubMed
Zhang J, Delzell E, Curtis JR, Hooven F, Gehlbach SH, Anderson FA Jr, Saag KG. Use of
pharmacologic agents for the primary prevention of osteoporosis among older women with
low bone mass. Osteoporos Int. 2014 Jan;25(1):317-24. doi: 10.1007/s00198-013-2444-0.
Epub 2013 Aug 28. PubMed PMID: 23982799. View Abstract at PubMed

Question 52 of 100

A B

52
Adult spine self-scored self-assessment examination AAOS 2015

Figures 52a and 52b are the radiographs of a patient who was involved in a
motor vehicle collision. He was wearing his seat belt and is now complaining
of midthoracic back pain. Radiographs in the emergency department do not
reveal a fracture. What is the most appropriate next step?

1- Nonsteroidal medication and follow up as needed


2- Repeat radiographs in 1 week
3- MR imaging of the thoracic spine
4- Flexion-extension radiographs

PREFERRED RESPONSE: 3- MR imaging of the thoracic spine

DISCUSSION

Ankylosing spinal disorders, including ankylosing spondylitis and diffuse


idiopathic skeletal hyperostosis, are conditions that make the spine rigid and
at risk for 3-column unstable fractures. Spinal fractures in these patients pose
high risk for complications and death and patients should be counseled and
observed closely. Mortality strongly correlates with older age and increased
number of comorbidities.
These spine fractures often are not seen at the time of initial evaluation, and
a delay in diagnosis can occur in up to 19% of cases. This is particularly
common in the setting of non- or minimally displaced fractures following minor
injuries. A delayed diagnosis can lead to displacement of a previously
nondisplaced fracture that can incur a high neurologic injury risk. Advanced
imaging with a CT scan or MRI should be obtained for patients with ankylosing
spinal disorders even when minor injuries occur. Although bracing and
observation can be used, posterior multilevel spinal instrumentation is
typically required to obtain adequate spinal stabilization.
The radiographs show an osteopenic ankylosed thoracic spine; the
anteroposterior radiograph clearly shows fusion of the sacroiliac joints.
Recognition of these radiographic findings is important when evaluating
patients after an injury.

RECOMMENDED READINGS

Caron T, Bransford R, Nguyen Q, Agel J, Chapman J, Bellabarba C. Spine fractures in patients


with ankylosing spinal disorders. Spine (Phila Pa 1976). 2010 May 15;35(11):E458-64. doi:
10.1097/BRS.0b013e3181cc764f. PubMed PMID: 20421858. View Abstract at PubMed

53
Adult spine self-scored self-assessment examination AAOS 2015

Hendrix RW, Melany M, Miller F, Rogers LF. Fracture of the spine in patients with ankylosis
due to diffuse skeletal hyperostosis: clinical and imaging findings. AJR Am J Roentgenol. 1994
Apr;162(4):899-904. PubMed PMID: 8141015. View Abstract at PubMed

Question 53 of 100

A 70-year-old otherwise healthy woman is undergoing correction of thoracic


kyphosis with spinal cord compression via a posterior approach. She receives
an inhaled anesthetic, her mean arterial pressure is 93 mm Hg, and her core
temperature is 37.2°C. Her spinal cord is being monitored with
somatosensory-evoked potentials and transcranial motor-evoked potentials.
She has very low amplitude and increased latency as detected by
neurophysiologic monitoring. Which action should be taken?

1- A Stagnara wake-up test should be performed immediately.


2- The patient's mean arterial pressure should be increased immediately.
3- The anesthesia team should use propofol instead of isoflurane.
4- The patient's core temperature should be decreased to 35°C.

PREFERRED RESPONSE: 3- The anesthesia team should use propofol instead


of isoflurane.

DISCUSSION

The use of halogenated inhaled anesthetic agents has been shown to abrogate
the signals detected during neurophysiologic monitoring. Intravenous agents
such as propofol should be used in favor of inhaled agents such as isoflurane
and nitrous oxide. Neurophysiologic signals also can be dampened when
hypotension and hypothermia are issues. In this case, the patient's mean
arterial pressure and body temperature are within the range at which spinal
cord blood flow and neurophysiologic monitoring may be optimized. A
Stagnara wake-up test may be useful, but it poses risk. If there is another
explanation for the patient's lack of signal, this test is not necessary.

RECOMMENDED READINGS

Devlin VJ, Schwartz DM. Intraoperative neurophysiologic monitoring during spinal surgery. J
Am Acad Orthop Surg. 2007 Sep;15(9):549-60. Review. PubMed PMID: 17761611. View
Abstract at PubMed

54
Adult spine self-scored self-assessment examination AAOS 2015

Chen Z. The effects of isoflurane and propofol on intraoperative neurophysiological monitoring


during spinal surgery. J Clin Monit Comput. 2004 Aug;18(4):303-8. PubMed PMID: 15779842.
View Abstract at PubMed

Question 54 of 100

A B

Figures 54a and 54b are the radiographs of a 21-year-old man who has a long
history of thoracic back pain. His lumbar spine is asymptomatic. He has failed
prolonged nonsurgical treatment. Surgical correction should consist of

1- anterior release and fusion at T4-10.


2- posterior instrumentation and fusion at T9-L5.
3- posterior instrumentation and fusion at T2-L2.
4- posterior instrumentation and fusion at T6-pelvis.

PREFERRED RESPONSE: 3- posterior instrumentation and fusion at T2-L2.

55
Adult spine self-scored self-assessment examination AAOS 2015

DISCUSSION

When planning surgical intervention for Scheuermann kyphosis, it is


imperative that the instrumentation and fusion extend across the entirety of
the deformity. Distally, this means extending across the first lordotic disk
space. In this scenario, this disk is the L1-L2 disk, which means the fusion
needs to extend to L2. Shorter and longer fusions are not necessary or
appropriate.

RECOMMENDED READINGS

Denis F, Sun EC, Winter RB. Incidence and risk factors for proximal and distal junctional
kyphosis following surgical treatment for Scheuermann kyphosis: minimum five-year follow-
up. Spine (Phila Pa 1976). 2009 Sep 15;34(20):E729-34. PubMed PMID: 19752692. View
Abstract at PubMed
Macagno AE, O'Brien MF. Thoracic and thoracolumbar kyphosis in adults. Spine (Phila Pa
1976). 2006 Sep 1;31(19 Suppl):S161-70. Review. PubMed PMID: 16946634. View Abstract
at PubMed

CLINICAL SITUATION FOR QUESTIONS 55 THROUGH 59

A 60-year-old woman has severe neck and back pain. She is relatively healthy
but has diabetes and neuropathy involving her lower extremities. Her body
mass index is 38. She has a history of spinal fusion performed by your
colleague 3 years ago. At that time, she was treated for degenerative scoliosis
of the lumbar spine with concomitant spinal stenosis. A 360-degree fusion was
performed from L4-S1 with a posterior decompression from L2-S1 and a
posterior instrumented fusion from T3 to the pelvis. On examination, she has
reproducible pain and a visible kyphosis in the periscapular region. Neurologic
examination findings are within normal limits, with the exception of lower-
extremity dysesthesias related to her neuropathy. The patient states that she
has been having progressive difficulty dressing herself and taking care of her
apartment for several months. Plain radiographs and a standing scoliosis
series demonstrate a solid fusion from the sacrum to T3 without evidence of
hardware failure. There is focal collapse of the T2-T3 disk space and a proximal
kyphosis involving the T2 vertebrae that is indicative of disk and ligamentous
failure.

Question 55 of 100

56
Adult spine self-scored self-assessment examination AAOS 2015

The patient asks why the proximal kyphosis occurred. You indicate that she
has several risk factors for this condition, including

1- her age, fusion to the sacrum, and upper instrumented vertebra at T3.
2- her age, 360-degree fusion, and obesity.
3- diabetes, history of neuropathy, and pelvic instrumentation.
4- upper instrumented vertebra at T3, obesity, and diabetes.

PREFERRED RESPONSE: 1- her age, fusion to the sacrum, and upper


instrumented vertebra at T3.

Question 56 of 100

Considering her current symptoms, history, and examination findings, you


believe that the most efficacious means with which to address this patient's
junctional kyphosis is

1- cervico-thoracic-lumbosacral orthosis to immobilize the kyphosed segment.


2- removal of the posterior instrumentation because she has had a successful
fusion.
3- anteroposterior instrumented fusion at T2-3 to address the collapsed
interspace.
4- revision posterior instrumentation with extension of the fusion to T1 to span
the area of junctional kyphosis.

PREFERRED RESPONSE: 4- revision posterior instrumentation with extension


of the fusion to T1 to span the area of junctional kyphosis.

Question 57 of 100

What are this patient's risk factors for postsurgical infection?

1- Presence of posterior spinal instrumentation


2- Anterior approach performed during the index procedure
3- Diabetes and obesity
4- Surgery performed in the thoracic spine

57
Adult spine self-scored self-assessment examination AAOS 2015

PREFERRED RESPONSE: 3- Diabetes and obesity

Question 58 of 100

Which complication is most frequently encountered after revision surgery to


treat junctional kyphosis?

1- Deep surgical-site infection


2- Pseudoarthrosis
3- Neurological injury
4- Adjacent segment degeneration

PREFERRED RESPONSE: 4- Adjacent segment degeneration

Question 59 of 100

During the revision surgical procedure, thoracic pedicle screws are placed.
Following placement, triggered electromyography (EMG) is performed by
stimulating the pedicle screw heads. During testing the right T2 pedicle screw
head returns a threshold of 2 mA. What does this reading indicate?

1- The right T2 pedicle screw is well seated within the pedicle.


2- The right T2 pedicle screw has breached the pedicle wall and has violated
the costovertebral junction.
3- There is a breach in the right T2 pedicle wall, but the screw is not in
contact with a neural structure.
4- There is a breach in the right T2 pedicle and the screw is in contact with a
neural structure.

PREFERRED RESPONSE: 4- There is a breach in the right T2 pedicle and the


screw is in contact with a neural structure.

DISCUSSION

This patient has developed a proximal junctional kyphosis following a long


posterior fusion performed for treatment of degenerative lumbar scoliosis.
Risk factors for proximal junctional kyphosis in the setting of instrumented
fusions performed for degenerative scoliosis include advanced age, 360-

58
Adult spine self-scored self-assessment examination AAOS 2015

degree fusions, extension of fusion constructs to the sacrum, and upper


instrumented vertebrae at the level of T1-3. The patient's junctional kyphosis
is attributable to failure of the disk and ligamentous structures at T2-3 and
would be graded as type I using the classification system of Yagi and
associates. Most symptomatic proximal junctional kyphoses are treated with
posterior extension of the fusion construct to a more proximal level spanning
the kyphosed region. The use of orthoses or simple removal of instrumentation
is unlikely to substantively impact symptoms or address the proximal
kyphosis. Diabetes and obesity are known factors that increase risk for
postsurgical infection following a spine fusion procedure. The most frequent
complication following revision surgery for proximal junctional kyphosis,
however, is the need for further surgery. In the study by Yagi and associates,
48% of patients who underwent revision surgery for proximal kyphosis
developed further adjacent segment degeneration. In the proximal thoracic
spine, where the pedicle may be narrow, triggered EMG testing of inserted
pedicle screws may be used to assess for violation of the pedicle wall during
insertion. In the setting of triggered EMG, thresholds exceeding 10 mA
indicate a well-placed pedicle screw. Thresholds lower than 4 mA to 6 mA
indicate that a screw is directly contacting a neural structure. Thresholds
between 9 mA and 10 mA suggest that a breach of the pedicle may be present,
but the screw is not contacting a neural structure.

RECOMMENDED READINGS

Yagi M, Rahm M, Gaines R, Maziad A, Ross T, Kim HJ, Kebaish K, Boachie-Adjei O; Complex
Spine Study Group. Characterization and surgical outcomes of proximal junctional failure in
surgically treated patients with adult spinal deformity.Spine (Phila Pa 1976). 2014 May
1;39(10):E607-14. doi: 10.1097/BRS.0000000000000266. PubMed PMID: 24525992. View
Abstract at PubMed
Cammarata M, Aubin CÉ, Wang X, Mac-Thiong JM. Biomechanical risk factors for proximal
junctional kyphosis: a detailed numerical analysis of surgical instrumentation variables. Spine
(Phila Pa 1976). 2014 Apr 15;39(8):E500-7. doi: 10.1097/BRS.0000000000000222. PubMed
PMID: 24480964. View Abstract at PubMed
Kim HJ, Lenke LG, Shaffrey CI, Van Alstyne EM, Skelly AC. Proximal junctional kyphosis as a
distinct form of adjacent segment pathology after spinal deformity surgery: a systematic
review. Spine (Phila Pa 1976). 2012 Oct 15;37(22 Suppl):S144-64. doi:
10.1097/BRS.0b013e31826d611b. PubMed PMID: 22885829. View Abstract at PubMed
Schoenfeld AJ, Carey PA, Cleveland AW 3rd, Bader JO, Bono CM. Patient factors,
comorbidities, and surgical characteristics that increase mortality and complication risk after
spinal arthrodesis: a prognostic study based on 5,887 patients. Spine J. 2013
Oct;13(10):1171-9. doi: 10.1016/j.spinee.2013.02.071. Epub 2013 Apr 9. PubMed PMID:
23578986. View Abstract at PubMed
Schoenfeld AJ. Spine infections. In: Cannada L, ed. Orthopaedic Knowledge Update 11.
Rosemont, IL: American Academy of Orthopaedic Surgeons, 2014: 737-747.

59
Adult spine self-scored self-assessment examination AAOS 2015

Schoenfeld AJ, Bono CM. Instrumented lumbar fusion. In: Flatow E, Colvin AC, eds. Atlas of
Essential Orthopaedic Procedures. Rosemont, IL: American Academy of Orthopaedic
Surgeons, 2013: 611-616.

Question 60 of 100

A B

Figures 60a and 60b are the postmyelography CT images of a 62-year-old


man who has had low-back and bilateral lower-extremity pain. His pain began
approximately 1 year ago and there was no precipitating event. The location
of the pain is in the lower lumbar region in the midline and it radiates into the
buttocks and thighs bilaterally. The pain is exacerbated by standing and
walking and relieved by forward spine flexion. He reports no pain while sitting
or lying supine. Upright radiographs demonstrate 4 mm of anterolisthesis of
L4 on L5. What is the most appropriate surgical procedure?

1- Microdiskectomy
2- Posterior decompression
3- Posterior decompression and fusion
4- Anterior lumbar interbody fusion

PREFERRED RESPONSE: 3- Posterior decompression and fusion

60
Adult spine self-scored self-assessment examination AAOS 2015

DISCUSSION

This patient radiographically has L4-5 degenerative stenosis and a low-grade


spondylolisthesis. The axial images demonstrate a gap in the facet joint, more
on the left than the right, which is highly suggestive of dynamic instability.
Thus, this patient would most benefit from a laminectomy and fusion of L4-5.
A microdiskectomy would not be helpful as this patient does not have a disk
herniation. Patients undergoing posterior decompression alone experience an
unacceptably high rate of recurrent stenosis and/or progression of
spondylolisthesis. Early studies demonstrated that adding an intertransverse
process fusion to posterior decompression significantly improved clinical
outcomes. Anterior interbody fusion likely will not decompress the spinal canal
sufficiently, though there is some thought that indirect decompression can be
effective in some cases.

RECOMMENDED READINGS

Frymoyer JW. Degenerative Spondylolisthesis: Diagnosis and Treatment. J Am Acad Orthop


Surg. 1994 Jan;2(1):9-15. PubMed PMID: 10708989. Majid K, Fischgrund JS. Degenerative
lumbar spondylolisthesis: trends in management. J Am Acad Orthop Surg. 2008
Apr;16(4):208-15. Review. PubMed PMID: 18390483. View Abstract at PubMed
Carlisle E, Fischgrund JS. Lumbar spinal stenosis and degenerative spondylolisthesis. In:
Spivak JM, Connolly PJ, eds. Orthopaedic Knowledge Update: Spine 3. Rosemont, IL:
American Academy of Orthopaedic Surgeons; 2006: 299-317.
Puschak TJ, Sasso RC. Spondylolysis-spondylolisthesis. In: Vaccaro AR, ed. Orthopaedic
Knowledge Update 8. Rosemont, IL: American Academy of Orthopaedic Surgeons; 2005:553-
563.

Question 61 of 100

A 78-year-old man with ankylosing spondylitis sustains a minor fall. Shortly


afterward he experiences sudden worsening of his chronic back pain and is
brought to the emergency department by his caregiver. Radiographs and a CT
scan of the spine do not show a clear fracture. What is the most appropriate
next step?

1- Nonsteroidal anti-inflammatory drugs


2- MRI of the spine
3- CT myelography
4- Flexion-extension plain films

61
Adult spine self-scored self-assessment examination AAOS 2015

PREFERRED RESPONSE: 2- MRI of the spine

DISCUSSION

Patients with ankylosing spondylitis are at high risk for occult fractures after
low-energy injuries. Although radiographs and a CT scan do not demonstrate
a spinal fracture in this patient, high risk for an unstable occult fracture
necessitates further imaging with MRI to ensure that no fractures are missed.
Although a CT scan is typically the primary imaging modality for workup of
spine injuries in similar patients, CT and MRI complement each other and each
detects fractures that are missed using the other modality. A CT myelogram
might detect cord or root compression but would not aid in the diagnosis of
an occult fracture. Nonsteroidal anti-inflammatory drugs are first-line
treatment for idiopathic low-back pain. In a patient with ankylosing spondylitis
at high risk for fracture, further workup is needed to rule out an occult
fracture. Flexion and extension radiographs of the spine are inferior to MRI for
evaluating occult fractures and ligamentous injuries. The primary concern for
this patient remains an unstable spinal fracture, which necessitates an MRI of
the spine before initiating a workup for other possible causes of his back pain.

RECOMMENDED READINGS

Duane TM, Cross J, Scarcella N, Wolfe LG, Mayglothling J, Aboutanos MB, Whelan JF, Malhotra
AK, Ivatury RR. Flexion-extension cervical spine plain films compared with MRI in the
diagnosis of ligamentous injury. Am Surg. 2010 Jun;76(6):595-8. PubMed PMID: 20583514.
View Abstract at PubMed
Hitchon PW, From AM, Brenton MD, Glaser JA, Torner JC. Fractures of the thoracolumbar
spine complicating ankylosing spondylitis. J Neurosurg. 2002 Sep;97(2 Suppl):218-22.
PubMed PMID: 12296682. View Abstract at PubMed
Koivikko MP, Koskinen SK. MRI of cervical spine injuries complicating ankylosing spondylitis.
Skeletal Radiol. 2008 Sep;37(9):813-9. doi: 10.1007/s00256-008-0484-x. Epub 2008 Apr
18. PubMed PMID: 18421455. View Abstract at PubMed
Wang YF, Teng MM, Chang CY, Wu HT, Wang ST. Imaging manifestations of spinal fractures
in ankylosing spondylitis. AJNR Am J Neuroradiol. 2005 Sep;26(8):2067-76. PubMed PMID:
16155161. View Abstract at PubMed
Westerveld LA, Verlaan JJ, Oner FC. Spinal fractures in patients with ankylosing spinal
disorders: a systematic review of the literature on treatment, neurological status and
complications. Eur Spine J. 2009 Feb;18(2):145-56. doi: 10.1007/s00586-008-0764-0. Epub
2008 Sep 13. Review. PubMed PMID: 18791749. View Abstract at PubMed

62
Adult spine self-scored self-assessment examination AAOS 2015

Question 62 of 100

A B C

Figures 62a through 62c are the MR images and CT scan of a 65-year-old man
with a history of diabetes mellitus, hypertension, and smoking. He has a 6-
week history of increasing midback pain, lower extremity pain, and weakness.
What is the most likely diagnosis, and how should this diagnosis be confirmed?

1- Thoracic tumor; CT-guided biopsy


2- Thoracic tumor; 3-phase bone scan
3- Thoracic discitis and osteomyelitis; CT-guided biopsy
4- Thoracic discitis and osteomyelitis; tagged white blood cell scan

PREFERRED RESPONSE: 1- Thoracic tumor; CT-guided biopsy

DISCUSSION

The sagittal T2-weighted and axial T2-weighted images show a lesion within
the T8 vertebral body that involves the posterior elements. There is an
associated epidural component that results in compression of the spinal cord.
The sagittal reconstructed CT image shows a lytic lesion within the T8
vertebral body. This pattern of vertebral body involvement with preservation

63
Adult spine self-scored self-assessment examination AAOS 2015

of the adjacent disks and endplates in a 65-year-old patient is most


compatible with a diagnosis of a tumor. The most likely tumor is a metastatic
lesion. A CT-guided biopsy will confirm this diagnosis. Although thoracic
tuberculosis does not typically cross the disk space, the lack of an anterior
soft-tissue component decreases the likelihood of this diagnosis.

RECOMMENDED READINGS

Khanna AJ, Shindle MK, Wasserman BA, Gokaslan ZL, Gonzales RA, Buchowski JM, Riley LH
3rd. Use of magnetic resonance imaging in differentiating compartmental location of spinal
tumors. Am J Orthop (Belle Mead NJ). 2005 Oct;34(10):472-6. Review. PubMed PMID:
16304794. View Abstract at PubMed
White AP, Kwon BK, Lindskog DM, Friedlaender GE, Grauer JN. Metastatic disease of the spine.
J Am Acad Orthop Surg. 2006 Oct;14(11):587-98. Review. PubMed PMID: 17030592. View
Abstract at PubMed

Question 63 of 100

Misplaced pedicle screws are the most common complication associated with
pedicle screw instrumentation. Many physicians use continuous
electromyography (EMG) monitoring and direct stimulation of the screw to
produce a peripheral EMG response. Which EMG stimulation minimum
threshold indicates an accurately placed pedicle screw?

1- Higher than 20 mA
2- Higher than 15 mA
3- Higher than 10 mA
4- 5 to 10 mA

PREFERRED RESPONSE: 1- Higher than 20 mA

DISCUSSION

A threshold of 15 mA or higher provides 98% confidence in accuracy of pedicle


screw placement. If the EMG stimulation threshold is lower than 4 mA to 6
mA, the screw likely is in contact with the nerve root or dura. A threshold
lower than 8 mA to 10 mA is associated with a breach in the pedicle cortex.

64
Adult spine self-scored self-assessment examination AAOS 2015

RECOMMENDED READINGS

Shen FH, Shaffrey CI, eds. Arthritis and Arthroplasty: The Spine. Philadelphia, PA: Saunders;
2010:141.
Glassman SD, Dimar JR, Puno RM, Johnson JR, Shields CB, Linden RD. A prospective analysis
of intraoperative electromyographic monitoring of pedicle screw placement with computed
tomographic scan confirmation. Spine (Phila Pa 1976). 1995 Jun 15;20(12):1375-9. PubMed
PMID: 7676335. View Abstract at PubMed

Question 64 of 100

What is the most significant benefit of percutaneous transforaminal lumbar


interbody fusion (TLIF) vs open posterior lumbar interbody fusion (PLIF)?

1- Preservation of the paraspinal muscle sleeve


2- Higher fusion rate
3- More measurable intraoperative blood loss
4- Improved ability to evacuate the disk space

PREFERRED RESPONSE: 1- Preservation of the paraspinal muscle sleeve

DISCUSSION

Humphreys and associates in a retrospective review of TLIF vs PLIF found


fusion rates, surgical time, and length of hospital stay were similar with both
procedures. The only benefits associated with TLIF were less blood loss and
preservation of the paraspinal muscle sleeve. Manos and associates in a
cadaver study found no difference in the volume of disk material evacuated
or the area of endplate exposed in either procedure.

RECOMMENDED READINGS

Humphreys SC, Hodges SD, Patwardhan AG, Eck JC, Murphy RB, Covington LA. Comparison
of posterior and transforaminal approaches to lumbar interbody fusion. Spine (Phila Pa 1976).
2001 Mar 1;26(5):567-71. PubMed PMID: 11242386. View Abstract at PubMed
Manos R, Sukovich W, Weistroffer J: Transforaminal lumbar interbody fusion: Minimally
invasive versus open disc excision and endplate preparation. Presented at the 12th
International Meeting of Advanced Spine Techniques, Banff, Alberta, Canada, July 7-9, 2005.

65
Adult spine self-scored self-assessment examination AAOS 2015

Question 65 of 100

Figure 65 is the lumbar spine MR image of a 63-year-old woman who has a 3-


year history of increasingly bothersome back pain and bilateral buttock and
leg pain. She has performed 6 weeks of physical therapy, received epidural
injections, and experienced some good short-term results, but her leg pain
continues to worsen. What is the most appropriate course of treatment?

1- Anterior lumbar interbody fusion L4-5


2- L4-5 hemilaminotomy and diskectomy
3- Lumbar laminectomy L4-L5 posterior fusion
4- Lumbar laminectomy at L4-5

PREFERRED RESPONSE: 3- Lumbar laminectomy L4-L5 posterior fusion

DISCUSSION

This patient has symptoms consistent with neurogenic claudication secondary


to lumbar spinal stenosis and degenerative spondylolisthesis. Her symptoms
are chronic and she has undergone an appropriate course of nonsurgical care.
Nevertheless, her symptoms are worsening and surgical intervention is a

66
Adult spine self-scored self-assessment examination AAOS 2015

reasonable consideration. Studies have shown that patients with lumbar spinal
stenosis with associated degenerative spondylolisthesis benefit most from
decompression of the neural elements that are stenotic and subsequent fusion
across the degenerative slip. Anterior lumbar interbody fusion likely will not
address stenosis at the level of the slip and may not result in adequate
neurologic decompression. Partial laminotomy and diskectomy likely will not
provide adequate neural decompression because these procedures would only
address unilateral compression and this patient has bilateral issues. Lumbar
laminectomy without fusion could be performed but has been associated with
results inferior to lumbar laminectomy with fusion when addressing stenosis
with spondylolisthesis.

RECOMMENDED READINGS

Weinstein JN, Lurie JD, Tosteson TD, Zhao W, Blood EA, Tosteson AN, Birkmeyer N, Herkowitz
H, Longley M, Lenke L, Emery S, Hu SS. Surgical compared with nonoperative treatment for
lumbar degenerative spondylolisthesis. four-year results in the Spine Patient Outcomes
Research Trial (SPORT) randomized and observational cohorts. J Bone Joint Surg Am. 2009
Jun;91(6):1295-304. PubMed PMID: 19487505. View Abstract at PubMed
Herkowitz HN, Kurz LT. Degenerative lumbar spondylolisthesis with spinal stenosis. A
prospective study comparing decompression with decompression and intertransverse process
arthrodesis. J Bone Joint Surg Am. 1991 Jul;73(6):802-8. PubMed PMID: 2071615. View
Abstract at PubMed
Lombardi JS, Wiltse LL, Reynolds J, Widell EH, Spencer C 3rd. Treatment of degenerative
spondylolisthesis. Spine (Phila Pa 1976). 1985 Nov;10(9):821-7. PubMed PMID: 4089657.
View Abstract at PubMed

RESPONSES FOR QUESTIONS 66 THROUGH 69

1- Deep surgical-site infection


2- Adjacent segment degeneration
3- Pressure ulcers
4- Iatrogenic neurologic injury
5- Incidental durotomy
6- Hardware failure
Match the frequently encountered complication listed above with the
appropriate clinical scenario below.

Question 66 of 100

67
Adult spine self-scored self-assessment examination AAOS 2015

A 63-year-old woman who was treated with decompression and instrumented


L4-L5 interbody fusion for degenerative spondylolisthesis demonstrates solid
fusion on radiographs.

1- Deep surgical-site infection


2- Adjacent segment degeneration
3- Pressure ulcers
4- Iatrogenic neurologic injury
5- Incidental durotomy
6- Hardware failure

PREFERRED RESPONSE: 2- Adjacent segment degeneration

Question 67 of 100

A 28-year-old obtunded man with a T12 burst fracture is treated in a


thoracolumbosacral orthosis.

1- Deep surgical-site infection


2- Adjacent segment degeneration
3- Pressure ulcers
4- Iatrogenic neurologic injury
5- Incidental durotomy
6- Hardware failure

PREFERRED RESPONSE: 3- Pressure ulcers

Question 68 of 100

A 42-year-old man with an L1 burst fracture, no neurologic deficit, and


evidence of disruption of the posterior ligamentous complex as observed on
MRI is treated with percutaneous stabilization without fusion.

1- Deep surgical-site infection


2- Adjacent segment degeneration
3- Pressure ulcers
4- Iatrogenic neurologic injury

68
Adult spine self-scored self-assessment examination AAOS 2015

5- Incidental durotomy
6- Hardware failure

PREFERRED RESPONSE: 6- Hardware failure

Question 69 of 100

A 46-year-old man with a sacral chordoma is treated with sacrectomy.

1- Deep surgical-site infection


2- Adjacent segment degeneration
3- Pressure ulcers
4- Iatrogenic neurologic injury
5- Incidental durotomy
6- Hardware failure

PREFERRED RESPONSE: 1- Deep surgical-site infection

DISCUSSION

Complications frequently occur following treatment for spinal disorders.


Postsurgical morbidity may range from minor complications such as urinary
tract infection to death. Wound infections are among the most commonly
encountered complications following any surgical intervention; however, their
incidence after spine surgery has been reported to be in the range of only 1%
to 6%. Meanwhile, risk for adjacent segment degeneration, particularly
following a fusion procedure performed with an interbody device, has been
documented as high as 30%. Pressure ulcers can occur when spinal injuries
are treated with rigid orthoses, especially in individuals who are immobile.
Risk for pressure sores and decubiti is further elevated in patients who are
obtunded and cannot communicate regarding pain levels and their need to
mobilize. Hardware failure may occur following any instrumented spine
procedure, but risk for this complication is elevated in the event of
pseudarthrosis or in settings in which no fusion is performed.
In the recent work of Lee and associates, hardware failure was the only
complication encountered among patients who were treated with
percutaneous stabilization without fusion for thoracolumbar burst fractures.
Similarly, iatrogenic neurologic injury may transpire in the setting of any
spinal intervention (its incidence is rare). For example, in a large cohort of

69
Adult spine self-scored self-assessment examination AAOS 2015

more than 10000 patients undergoing surgical treatment for spondylolisthesis,


acute neurologic injury was reported in only 0.8% of cases. Postsurgical
wound infection, although relatively rare in cases of elective spine surgery, is
common following large tumor resection such as sacrectomy performed for
treatment of chordomas.

RECOMMENDED READINGS

Eismont FJ, Norton RP, Hirsch BP. Surgical management of lumbar degenerative
spondylolisthesis. J Am Acad Orthop Surg. 2014 Apr;22(4):203-13. doi: 10.5435/JAAOS-22-
04-203. Review. PubMed PMID: 24668350. View Abstract at PubMed
Bransford RJ, Morgan RA. Thoracolumbar trauma. In: Schmidt AH, Teague DC, eds.
Orthopaedic Knowledge Update: Trauma 4. Rosemont, IL: American Academy of Orthopaedic
Surgeons; 2010:373-385.
Lee JK, Jang JW, Kim TW, Kim TS, Kim SH, Moon SJ. Percutaneous short-segment pedicle
screw placement without fusion in the treatment of thoracolumbar burst fractures: is it
effective? comparative study with open short-segment pedicle screw fixation with
posterolateral fusion. Acta Neurochir (Wien). 2013 Dec;155(12):2305-12; discussion 2312.
doi: 10.1007/s00701-013-1859-x. Epub 2013 Sep 10. PubMed PMID: 24018981. View
Abstract at PubMed
Schoenfeld AJ, Carey PA, Cleveland AW 3rd, Bader JO, Bono CM. Patient factors,
comorbidities, and surgical characteristics that increase mortality and complication risk after
spinal arthrodesis: a prognostic study based on 5,887 patients. Spine J. 2013
Oct;13(10):1171-9. doi: 10.1016/j.spinee.2013.02.071. Epub 2013 Apr 9. PubMed PMID:
23578986. View Abstract at PubMed
Sansur CA, Reames DL, Smith JS, Hamilton DK, Berven SH, Broadstone PA, Choma TJ, Goytan
MJ, Noordeen HH, Knapp DR Jr, Hart RA, Zeller RD, Donaldson WF 3rd, Polly DW Jr, Perra JH,
Boachie-Adjei O, Shaffrey CI. Morbidity and mortality in the surgical treatment of 10,242
adults with spondylolisthesis. J Neurosurg Spine. 2010 Nov;13(5):589-93. doi:
10.3171/2010.5.SPINE09529. PubMed PMID: 21039149. View Abstract at PubMed
Dubory A, Missenard G, Lambert B, Court C. "En bloc" resection of sacral chordomas by
combined anterior and posterior surgical approach: a monocentric retrospective review about
29 cases. Eur Spine J. 2014 Sep;23(9):1940-8. doi: 10.1007/s00586-014-3196-z. Epub 2014
Jan 28. PubMed PMID: 24469886. View Abstract at PubMed

Question 70 of 100

An awake and alert patient with neck pain arrives at the emergency
department after an automobile crash. Upon examination he is weak in the
left deltoid and biceps muscles (3/5 strength). CT scans performed 2 hours
after admission are shown in Figures 70a and 70b. His weakness deteriorates

70
Adult spine self-scored self-assessment examination AAOS 2015

to 1/5 strength in the upper and lower extremities. What is the most
appropriate treatment?

A B

1- Immediate closed reduction in the intensive care unit while the patient is
awake
2- Posterior spinal laminectomy and fusion with instrumentation
3- Anterior cervical diskectomy, corpectomy, and plating
4- High-dose methylprednisolone

PREFERRED RESPONSE: 1- Immediate closed reduction in the intensive care


unit while the patient is awake

DISCUSSION

Facet subluxation reduction may be performed in awake patients. Posterior


spinal laminectomy and fusion can result in worsening neurologic status and
is not recommended in this setting. Generally, corpectomy in the setting of
facet subluxation is not recommended because it does not facilitate reduction
or fully alleviate spinal cord compression. High-dose steroid use is not
supported by current literature.

RECOMMENDED READINGS

71
Adult spine self-scored self-assessment examination AAOS 2015

Fehlings MG, Perrin RG. The timing of surgical intervention in the treatment of spinal cord
injury: a systematic review of recent clinical evidence. Spine (Phila Pa 1976). 2006 May
15;31(11 Suppl):S28-35; discussion S36. Review. PubMed PMID: 16685233. View Abstract
at PubMed
Lee AS, MacLean JC, Newton DA. Rapid traction for reduction of cervical spine dislocations. J
Bone Joint Surg Br. 1994 May;76(3):352-6. PubMed PMID: 8175833.View Abstract at PubMed

Question 71 of 100

Pain emanating from the sacroiliac (SI) joint is best identified by which of the
following maneuvers?

1- Reproduction of pain with the Gaenslen test


2- Reproduction of pain with the SI joint compression test
3- Presence of bone marrow edema on short tau inversion recovery
sequence of MR images.
4- More than 75% pain reduction following fluoroscopically guided SI joint
injection.

PREFERRED RESPONSE: 4- More than 75% pain reduction following


fluoroscopically guided SI joint injection.

DISCUSSION

Though no gold standard exists, a reduction of concordant pain by at least 75


to 80% following an intra-articular, image-guided anesthetic injection is
considered to be the most reliable method of identifying the SI joint as the
cause of a patient's pain. Although provocation tests including the Gaenslen
test, the compression test, thigh thrust, and Yeoman test are commonly used
and can be helpful in diagnosing non-specific SI joint pain, individually they
are not as reliable as the response to a diagnostic, anesthetic injection. Of
note, the combination of all 4 manuevers has proven to be more useful than
any one individual test. An MRI of the SI joint showing bony erosion and bone
marrow edema suggests inflammatory arthritis and may not necessarily be
associated with pain.

RECOMMENDED READINGS

72
Adult spine self-scored self-assessment examination AAOS 2015

Hancock MJ, Maher CG, Latimer J, Spindler MF, McAuley JH, Laslett M, Bogduk N. Systematic
review of tests to identify the disc, SIJ or facet joint as the source of low back pain. Eur Spine
J. 2007 Oct;16(10):1539-50. Epub 2007 Jun 14. PubMed PMID: 17566796. View Abstract at
PubMed
Visser LH, Nijssen PG, Tijssen CC, van Middendorp JJ, Schieving J. Sciatica-like symptoms
and the sacroiliac joint: clinical features and differential diagnosis. Eur Spine J. 2013
Jul;22(7):1657-64. doi: 10.1007/s00586-013-2660-5. Epub 2013 Mar 2. PubMed PMID:
23455949. View Abstract at PubMed
Weber U, Zubler V, Pedersen SJ, Rufibach K, Lambert RG, Chan SM, Ostergaard M,
Maksymowych WP. Development and validation of a magnetic resonance imaging reference
criterion for defining a positive sacroiliac joint magnetic resonance imaging finding in
spondyloarthritis. Arthritis Care Res (Hoboken). 2013 Jun;65(6):977-85. doi:
10.1002/acr.21893. PubMed PMID: 23203670. View Abstract at PubMed

CLINICAL SITUATION FOR QUESTION 72 THROUGH 75

Figures 72a through 72c are the sagittal CT scan and thoracic MR images of a
52-year-old woman with a history of pancreatic neuroendocrine tumor who
has severe upper thoracic back pain despite receiving aggressive oral pain
treatment. She has metastases in her liver, adrenal glands, and abdominal
mesentery. The thoracic disease has been treated with conventional radiation.
She continues to work her part-time job without experiencing signs or
symptoms of myelopathy.

A B C

73
Adult spine self-scored self-assessment examination AAOS 2015

Question 72 of 100

What is the best next step in managing her pain?

1- Additional conventional radiation


2- En bloc spondylectomy
3- Palliative spinal cord decompression and stabilization
4- Epidural steroid injection

PREFERRED RESPONSE: 3- Palliative spinal cord decompression and


stabilization

Question 73 of 100

What is the most appropriate systemic therapy for this patient in the short
term?

1- Bisphosphonate therapy
2- Monoclonal antibody against receptor activator of nuclear factor kappa
beta ligand
3- Doxorubicin
4- Dexamethasone

PREFERRED RESPONSE: 4- Dexamethasone

Question 74 of 100

If surgery is chosen, what is the optimum procedure?

1- Transthoracic corpectomy followed by posterior instrumentation


2- Percutaneous pedicle screw stabilization
3- Costotransversectomy with posterior instrumentation
4- Posterior total en bloc spondylectomy

PREFERRED RESPONSE: 3- Costotransversectomy with posterior


instrumentation

74
Adult spine self-scored self-assessment examination AAOS 2015

Question 75 of 100

What is the goal of surgical treatment in this scenario?

1- To cure cancer
2- To prolong life
3- To relieve pain
4- To reverse neurologic symptoms

PREFERRED RESPONSE: 3- To relieve pain

DISCUSSION

This patient has a metastatic neuroendocrine tumor. Surgical treatment


should prioritize palliation of her symptoms. She has high-grade spinal cord
compression without neurologic signs or symptoms. Steroids are beneficial for
patients with high-grade spinal cord compression caused by tumors, and these
drugs should be administered in the acute setting. This patient was
appropriately initially treated with conventional radiation. However, she is not
a candidate for further radiation because of spinal cord tolerance limits and
insufficient clearance between the tumor and spinal cord. Consequently,
stereotactic radiation is not an option.
The goal of surgical treatment of this tumor should be palliation of her
symptoms rather than cure. A costotransversectomy approach offers the
advantage of ventral and dorsal spinal cord access, which is necessary in this
case. A sternotomy or transthoracic approach would offer ventral access, but
dorsal access would be less than optimal.

RECOMMENDED READINGS

Rose PS, Buchowski JM. Metastatic disease in the thoracic and lumbar spine: evaluation and
management. J Am Acad Orthop Surg. 2011 Jan;19(1):37-48. Review. PubMed PMID:
21205766.View Abstract at PubMed
Rades D, Abrahm JL. The role of radiotherapy for metastatic epidural spinal cord compression.
Nat Rev Clin Oncol. 2010 Oct;7(10):590-8. doi: 10.1038/nrclinonc.2010.137. Epub 2010 Aug
31. Review. PubMed PMID: 20808299. View Abstract at PubMed

Question 76 of 100

75
Adult spine self-scored self-assessment examination AAOS 2015

An obtunded 80-year-old man was found alone in his apartment after an


apparent fall. A CT scan performed in the emergency department shows that
he has an extensile injury of an ankylosed cervical spine. The fracture extends
across the ossified C5-C6 disk space and into the lamina of C5. There is 1.5
cm of widening between the C5 and C6 vertebrae anteriorly. The patient's
family asks you about the long-term impact of the fracture on his functional
capacity and survival. You advise them that patients with fractures of the
cervical spine with ankylosing conditions have

1- An unknown prognosis until they can participate in an examination.


2- Higher rates of neurologic deficit and mortality than other same-age people.
3-Higher rates of neurologic deficit than other same-age people, but
comparable mortality rates.
4- Similar rates of neurologic deficit and mortality as other same-age people.

PREFERRED RESPONSE: 2- higher rates of neurologic deficit and mortality


than other same-age people.

DISCUSSION

Several studies have found that rates of neurologic deficit and mortality are
higher for patients with ankylosing spondylitis and a spinal fracture than for
age-matched controls. The 2011 work of Schoenfeld and associates, which
directly compared patients with cervical fractures in ankylosed spines to age-
and sex-matched controls who also had cervical fractures but no ankylosing
condition, demonstrated that those with ankylosing spondylitis were at
elevated risk for mortality for up to 2 years after sustaining a fracture. In a
study by Westerveld and associates, the rate of neurologic deficit among
patients with ankylosing spondylitis and a spinal fracture was 57.1%
compared to 12.6% among controls.

RECOMMENDED READINGS

Westerveld LA, van Bemmel JC, Dhert WJ, Oner FC, Verlaan JJ. Clinical outcome after
traumatic spinal fractures in patients with ankylosing spinal disorders compared with control
patients. Spine J. 2014 May 1;14(5):729-40. doi: 10.1016/j.spinee.2013.06.038. Epub 2013
Aug 27. PubMed PMID: 23992936. View Abstract at PubMed
Schoenfeld AJ, Harris MB, McGuire KJ, Warholic N, Wood KB, Bono CM. Mortality in elderly
patients with hyperostotic disease of the cervical spine after fracture: an age- and sex-
matched study. Spine J. 2011 Apr;11(4):257-64. doi: 10.1016/j.spinee.2011.01.018. Epub
2011 Mar 5. PubMed PMID: 21377938. View Abstract at PubMed

76
Adult spine self-scored self-assessment examination AAOS 2015

Question 77 of 100

Which clinical signs are consistent with the diagnosis of cauda equina
syndrome?

1- Saddle anesthesia, hyperreflexia, quadriceps weakness


2- Decreased motor function in the quadriceps and gastrocnemius,
hyperreflexia,? and saddle anesthesia
3- More than 5 beats of clonus, saddle anesthesia, and bowel incontinence
4- Urinary retention, saddle anesthesia, and extensor hallucis longus
weakness

PREFERRED RESPONSE: 4- Urinary retention, saddle anesthesia, and extensor


hallucis longus weakness

DISCUSSION

Cauda equina syndrome is a lower-motor neuron deficit. Hyperreflexia, clonus,


and other upper-motor neuron findings would not be seen. Saddle anesthesia,
motor weakness, and neurogenic bladder are elements critical to the diagnosis
of cauda equina syndrome.

RECOMMENDED READINGS

Kostuik JP, Harrington I, Alexander D, Rand W, Evans D. Cauda equina syndrome and lumbar
disc herniation. J Bone Joint Surg Am. 1986 Mar;68(3):386-91. PubMed PMID: 2936744. View
Abstract at PubMed
Spector LR, Madigan L, Rhyne A, Darden B 2nd, Kim D. Cauda equina syndrome. J Am Acad
Orthop Surg. 2008 Aug;16(8):471-9. Review. PubMed PMID: 18664636. View Abstract at
PubMed

CLINICAL SITUATION FOR QUESTIONS 78 THROUGH 80

Figures 78a and 78b are the axial and sagittal MR images of an otherwise
healthy 24-year-old woman who has had 8 weeks of severe leg pain without
weakness.

77
Adult spine self-scored self-assessment examination AAOS 2015

A B

Question 78 of 100

Based on this patient's MR images, at which location would you expect to find
altered sensation?

1- Medial side of left leg


2- Medial side of left foot
3- Dorsum of left foot
4- Lateral aspect of left foot

PREFERRED RESPONSE: 4- Lateral aspect of left foot

Question 79 of 100

A minimally invasive diskectomy technique poses potential for

1- increased dural tear risk.


2- a lower success rate for relief of leg pain.
3- a lower overall complication rate.
4- a higher rate of recurrent disk herniation.

78
Adult spine self-scored self-assessment examination AAOS 2015

PREFERRED RESPONSE: 1- increased dural tear risk.

Question 80 of 100

A diskectomy is performed in which the disk space is not aggressively


debrided. When compared to techniques that involve aggressive debridement
of the disk space, this results in

1- less intraoperative blood loss.


2- an increased rate of recurrent disk herniation.
3- a shorter length of hospital stay.
4- a higher rate of surgical complications.

PREFERRED RESPONSE: 2- an increased rate of recurrent disk herniation.

DISCUSSION

This patient has disk herniation at the left L5-S1 level. This will generally affect
the traversing S1 nerve. The S1 dermatome is on the lateral aspect and sole
of the foot.
Surgical treatment generally involves a diskectomy with removal of the
herniated fragment. This can be performed via a conventional open approach
or minimally invasive endoscopic technique. Several recent meta-analyses
have demonstrated equivalent outcomes with regard to leg pain and clinical
outcomes. Although minimally invasive techniques have been associated with
an increased rate of dural tear, the overall complication rate between the 2
techniques is not significantly different. Several studies have demonstrated a
substantial learning curve associated with minimally invasive techniques, and
the rate of complications decreases significantly with surgeon experience.
When performing a diskectomy, the herniated fragment alone can be removed
(sequestrectomy) or some of the disk that remains in the disk space can be
removed (complete diskectomy). Studies have shown no change in surgical
time, blood loss, length of stay, or surgical complications when performing a
sequestrectomy (compared to a more complete diskectomy). A
sequestrectomy is associated with a higher rate of recurrent disk herniation
at the surgical level.

RECOMMENDED READINGS

Kamper SJ, Ostelo RW, Rubinstein SM, Nellensteijn JM, Peul WC, Arts MP, van Tulder MW.
Minimally invasive surgery for lumbar disc herniation: a systematic review and meta-analysis.

79
Adult spine self-scored self-assessment examination AAOS 2015

Eur Spine J. 2014 May;23(5):1021-43. doi: 10.1007/s00586-013-3161-2. Epub 2014 Jan 18.
PubMed PMID: 24442183. View Abstract at PubMed
Dasenbrock HH, Juraschek SP, Schultz LR, Witham TF, Sciubba DM, Wolinsky JP, Gokaslan
ZL, Bydon A. The efficacy of minimally invasive discectomy compared with open discectomy:
a meta-analysis of prospective randomized controlled trials. J Neurosurg Spine. 2012
May;16(5):452-62. doi: 10.3171/2012.1.SPINE11404. Epub 2012 Mar 9. PubMed PMID:
22404142. View Abstract at PubMed
Lee P, Liu JC, Fessler RG. Perioperative results following open and minimally invasive single-
level lumbar discectomy. J Clin Neurosci. 2011 Dec;18(12):1667-70. doi:
10.1016/j.jocn.2011.04.004. Epub 2011 Sep 25. PubMed PMID: 21944927. View Abstract at
PubMed
Shamji MF, Bains I, Yong E, Sutherland G, Hurlbert RJ. Treatment of Herniated Lumbar Disk
by Sequestrectomy or Conventional Diskectomy. World Neurosurg. 2013 Feb 20. pii: S1878-
8750(13)00352-5. doi: 10.1016/j.wneu.2013.02.066. [Epub ahead of print] Review. PubMed
PMID: 23454687. View Abstract at PubMed
Wang H, Huang B, Li C, Zhang Z, Wang J, Zheng W, Zhou Y. Learning curve for percutaneous
endoscopic lumbar discectomy depending on the surgeon's training level of minimally invasive
spine surgery. Clin Neurol Neurosurg. 2013 Oct;115(10):1987-91. doi:
10.1016/j.clineuro.2013.06.008. Epub 2013 Jul 2. PubMed PMID: 23830496. View Abstract
at PubMed
Soliman J, Harvey A, Howes G, Seibly J, Dossey J, Nardone E. Limited microdiscectomy for
lumbar disk herniation: a retrospective long-term outcome analysis. J Spinal Disord Tech.
2014 Feb;27(1):E8-E13. doi: 10.1097/BSD.0b013e31828da8f1. PubMed PMID: 23563332.
View Abstract at PubMed
Radcliff K, Hilibrand A, Lurie JD, Tosteson TD, Delasotta L, Rihn J, Zhao W, Vaccaro A, Albert
TJ, Weinstein JN. The impact of epidural steroid injections on the outcomes of patients treated
for lumbar disc herniation: a subgroup analysis of the SPORT trial. J Bone Joint Surg Am.
2012 Aug 1;94(15):1353-8. doi: 10.2106/JBJS.K.00341. PubMed PMID: 22739998. View
Abstract at PubMed

Question 81 of 100

Figure 81 is a lateral thoracic spine radiograph of a 76-year-old man with a


history of ankylosing spondylitis who falls and strikes his back. He has
moderate thoracic discomfort. An initial examination does not reveal
neurologic deficits. He is discharged home that day, but returns 3 days later
with profound weakness in his legs. Which imaging study should have been
obtained at his initial presentation?

1- Plain radiographs of the lumbar spine


2- Anteroposterior radiograph of the pelvis
3- CT scan of the thoracic spine
4- MRI of the brain

80
Adult spine self-scored self-assessment examination AAOS 2015

PREFERRED RESPONSE: 3- CT scan of the thoracic spine

DISCUSSION

Patients with ankylosing spondylitis are at high risk for occult vertebral
fractures that are not readily detectable on radiographs. The treating surgeon
must have a high suspicion for fractures in these patients and pursue further
imaging of the spine with CT and (often) MRI. Even among patients who are
neurologically intact, fracture displacement and neurologic deterioration can
occur if fractures are not recognized early and appropriately stabilized.
Fractures in patients with ankylosing spondylitis are extremely unstable and
are associated with high risk for delayed neurological deterioration. Although
plain film imaging of the entire spine should be considered, occult fractures
can easily be missed. Imaging of the sacroiliac joints can be helpful to
establish the diagnosis of ankylosing spondylitis but would not identify an
occult fracture of the vertebra in this patient.

RECOMMENDED READINGS

Finkelstein JA, Chapman JR, Mirza S. Occult vertebral fractures in ankylosing spondylitis.
Spinal Cord. 1999 Jun;37(6):444-7. PubMed PMID: 10432265. View Abstract at PubMed
Harrop JS, Sharan A, Anderson G, Hillibrand AS, Albert TJ, Flanders A, Vaccaro AR. Failure of
standard imaging to detect a cervical fracture in a patient with ankylosing spondylitis. Spine
(Phila Pa 1976). 2005 Jul 15;30(14):E417-9. PubMed PMID: 16025019. View Abstract at
PubMed
Westerveld LA, Verlaan JJ, Oner FC. Spinal fractures in patients with ankylosing spinal
disorders: a systematic review of the literature on treatment, neurological status and

81
Adult spine self-scored self-assessment examination AAOS 2015

complications. Eur Spine J. 2009 Feb;18(2):145-56. doi: 10.1007/s00586-008-0764-0. Epub


2008 Sep 13. Review. PubMed PMID: 18791749. View Abstract at PubMed

Question 82 of 100

Which factor should most influence a patient's decision to have surgery for
adult scoliosis if he or she is younger than age 50?

1- Increasing coronal plane deformity


2- Increasing pain
3- Increasing sagittal balance
4- Invasiveness of the surgical procedure

PREFERRED RESPONSE: 1- Increasing coronal plane deformity

DISCUSSION

In a retrospective review of 137 patients treated surgically and 153 patients


treated nonsurgically for adult scoliosis, Bess and associates found that
surgical treatment for patients younger than 50 years of age was driven by
increased coronal plane deformity, and surgical treatment for older patients
was mandated by pain and disability. They also concluded that age,
comorbidities, and sagittal balance did not influence treatment decisions.

RECOMMENDED READINGS

Bess S, Boachie-Adjei O, Burton D, Cunningham M, Shaffrey C, Shelokov A, Hostin R, Schwab


F, Wood K, Akbarnia B; International Spine Study Group. Pain and disability determine
treatment modality for older patients with adult scoliosis, while deformity guides treatment
for younger patients. Spine (Phila Pa 1976). 2009 Sep 15;34(20):2186-90. PubMed PMID:
19752704.View Abstract at PubMed
Anderson DG, Albert T, Tannoury C. Adult scoliosis. In: Spivak JM, Connolly PJ, eds.
Orthopaedic Knowledge Update: Spine 3. Rosemont, IL: American Academy of Orthopaedic
Surgeons; 2006:331-338.

82
Adult spine self-scored self-assessment examination AAOS 2015

Question 83 of 100

Figure 83 is the CT scan of a 36-year-old man who fell from a roof. Eight hours
later at the emergency department he describes low-back pain with numbness
and weakness in his bilateral lower extremity. A neurologic examination
reveals 2/5 strength in his quadriceps and iliopsoas bilaterally, 2/5 strength
in his right anterior tibialis and gastrocsoleus, and 1/5 strength in his left
anterior tibialis and gastrocsoleus. Two hours later, strength in his lower
extremities has diminished markedly. What is the best next step?

1- Intravenous (IV) methylprednisolone with a 30-mg/kg loading dose


followed by continuous infusion of 5.4 mg/kg/hour for 24 hours
2- Immediate awake traction reduction
3- Emergent open reduction/decompression
4- Admission to the intensive care unit for fluid resuscitation followed by
reduction/decompression when stable

PREFERRED RESPONSE: 3- Emergent open reduction/decompression

DISCUSSION

83
Adult spine self-scored self-assessment examination AAOS 2015

Any progressive neurologic deficit requires emergent surgical intervention.


Lumbar injuries cannot be reliably reduced with traction. Although IV steroids
and management of mean arterial blood pressure are appropriate
interventions for injuries in the region of the conus medullaris, steroids are
only indicated when given within 8 hours of injury and are not appropriate as
a sole means of management for progressive neurologic deficit.

RECOMMENDED READINGS

Bono CM, Rinaldi MD. Thoracolumbar trauma. In: Spivak JM, Connolly PJ, eds. Orthopaedic
Knowledge Update: Spine 3. Rosemont, IL: American Academy of Orthopaedic Surgeons;
2006:201-216.
Mikles MR, Stchur RP, Graziano GP. Posterior instrumentation for thoracolumbar fractures. J
Am Acad Orthop Surg. 2004 Nov-Dec;12(6):424-35. Review. PubMed PMID: 15615508. View
Abstract at PubMed

Question 84 of 100

A 65-year-old woman undergoes a lumbar laminectomy for spinal stenosis at


the L3-L4 level. The surgery and postsurgical course are uncomplicated. Eight
weeks after surgery she has severe left anterior thigh, groin, and knee pain
with ambulation and standing. Which condition is the most likely cause of her
symptoms?

1- Epidural hematoma
2- Osteoarthritis of the hip
3- Miralgia paraesthetica
4- Facet joint pain

PREFERRED RESPONSE: 2- Osteoarthritis of the hip

DISCUSSION

Disorders of the hip can mimic and/or coexist with lumbar spine disorders.
The prevalence of hip pain lasting longer than 1 month in patients ages 65 to
74 years is 19%. There is often overlap between their respective signs and
symptoms. In a patient with failed back surgery syndrome, hip pathology may
have been present before back surgery and not recognized. Osteoarthritis of

84
Adult spine self-scored self-assessment examination AAOS 2015

the hip typically causes groin and anterior thigh pain. Meralgia paraesthetica
is more likely to manifest immediately after surgery. Trochanteric bursitis
usually affects the proximal lateral thigh and often can radiate to the distal
thigh. Facet joint pain causes low-back pain that can be referred to the gluteal
region. Epidural hematoma 6 weeks after surgery is highly unlikely.

RECOMMENDED READINGS

Bolt PM, Wahl MM, Schofferman J: The roles of the hip, spine, sacroiliac joint, and other
structures in patients with persistent pain after back surgery. Seminars in Spine surgery
2008;20:14-19.
Brown MD, Gomez-Marin O, Brookfield KF, Li PS. Differential diagnosis of hip disease versus
spine disease. Clin Orthop Relat Res. 2004 Feb;(419):280-4. PubMed PMID: 15021166. View
Abstract at PubMed

Question 85 of 100

Figures 85a through 85c are the sagittal and axial CT scans and sagittal T2
MR image of a 21-year-old man who was thrown from his motocross bike
earlier in the day. He now has significant low-back pain; however, he is
neurologically intact and has no trouble voiding urine. A standing plain
radiograph obtained the next day is shown in Figure 85d. Treatment should
involve

A B

85
Adult spine self-scored self-assessment examination AAOS 2015

C D

1- resumption of full activity as soon as tolerated.


2- a brace.
3- anterior stabilization and fusion.
4- posterior stabilization and fusion.

PREFERRED RESPONSE: 2- a brace.

DISCUSSION

Disruption of the posterior ligamentous complex is an important determinant


of the stability of a burst fracture. This patient is neurologically intact and his
MR images do not reveal posterior ligamentous complex (PLC) disruption. The
standing radiograph confirms that overall alignment is acceptably and
relatively preserved. Nonsurgical treatment with or without a brace is
acceptable in this scenario; however, the patient should not be cleared to
resume full activity until fracture healing, which may be as long as 3 months
after the date of injury. Anterior or posterior surgery should be reserved for
patients with PLC disruption, neurological injury, or, in some cases, multiple
trauma.

RECOMMENDED READINGS

86
Adult spine self-scored self-assessment examination AAOS 2015

Wood K, Buttermann G, Mehbod A, Garvey T, Jhanjee R, Sechriest V. Operative compared


with nonoperative treatment of a thoracolumbar burst fracture without neurological deficit. A
prospective, randomized study. J Bone Joint Surg Am. 2003 May;85-A(5):773-81. Erratum
in: J Bone Joint Surg Am. 2004 Jun;86-A(6):1283. Butterman, G [corrected to Buttermann,
G]. PubMed PMID: 12728024. View Abstract at PubMed
Vaccaro AR, Lehman RA Jr, Hurlbert RJ, Anderson PA, Harris M, Hedlund R, Harrop J, Dvorak
M, Wood K, Fehlings MG, Fisher C, Zeiller SC, Anderson DG, Bono CM, Stock GH, Brown AK,
Kuklo T, Oner FC. A new classification of thoracolumbar injuries: the importance of injury
morphology, the integrity of the posterior ligamentous complex, and neurologic status. Spine
(Phila Pa 1976). 2005 Oct 15;30(20):2325-33. PubMed PMID: 16227897. View Abstract at
PubMed
Vaccaro AR, Zeiller SC, Hulbert RJ, Anderson PA, Harris M, Hedlund R, Harrop J, Dvorak M,
Wood K, Fehlings MG, Fisher C, Lehman RA Jr, Anderson DG, Bono CM, Kuklo T, Oner FC. The
thoracolumbar injury severity score: a proposed treatment algorithm. J Spinal Disord Tech.
2005 Jun;18(3):209-15. PubMed PMID: 15905761.View Abstract at PubMed

RESPONSES FOR QUESTIONS 86 THROUGH 89

1- Proximal junctional kyphosis (PJK)


2- Adjacent segment degeneration
3- Intraoperative neurological injury
4- Postsurgical wound infection
Please select the complication listed above that most commonly is
associated with a clinical scenario described below.

Question 86 of 100

A 68-year-old undergoes surgery for adult scoliosis with sagittal imbalance


that necessitates a large kyphotic correction.

1- Proximal junctional kyphosis (PJK)


2- Adjacent segment degeneration
3- Intraoperative neurological injury
4- Postsurgical wound infection

PREFERRED RESPONSE: 1- Proximal junctional kyphosis (PJK)

Question 87 of 100

87
Adult spine self-scored self-assessment examination AAOS 2015

A 47-year-old man undergoes a 3-column osteotomy as part of scoliosis


surgery. During closure, somatosensory-evoked potentials decrease.

1- Proximal junctional kyphosis (PJK)


2- Adjacent segment degeneration
3- Intraoperative neurological injury
4- Postsurgical wound infection

PREFERRED RESPONSE: 3- Intraoperative neurological injury

Question 88 of 100

A 35-year-old undergoes an L4/L5 posterior fusion for spondylolisthesis.


Postsurgical radiographs reveal segmental kyphosis and overall sagittal
imbalance.

1- Proximal junctional kyphosis (PJK)


2- Adjacent segment degeneration
3- Intraoperative neurological injury
4- Postsurgical wound infection
PREFERRED RESPONSE: 2- Adjacent segment degeneration

Question 89 of 100

A 62-year-old woman undergoes a posterior laminectomy and fusion from L3


to S1. Postsurgical images show a pelvic incidence (PI) that is significantly
larger than the lumbar lordosis (LL).

1- Proximal junctional kyphosis (PJK)


2- Adjacent segment degeneration
3- Intraoperative neurological injury
4- Postsurgical wound infection

PREFERRED RESPONSE: 1- Proximal junctional kyphosis (PJK)

DISCUSSION

88
Adult spine self-scored self-assessment examination AAOS 2015

Sagittal balance is becoming increasingly important. PI should be roughly


equal to LL. A PI larger than the LL has been associated with a higher incidence
of PJK. PJK also is associated with larger sagittal balance corrections.
Additional risk factors include higher presurgical thoracic kyphosis, higher
postsurgical lordosis, osteoporosis, fusion below L2, and older (greater than
65 years old) patient age. The loss of somatosensory-evoked potentials during
surgery, especially during osteotomy correction, is concerning for neurological
injury and should be investigated. If no other cause for a change in signals
can be identified, the correction should be lessened to take pressure off of the
nerves.

RECOMMENDED READINGS

Ghobrial GM, Thakkar V, Andrews E, Lang M, Chitale A, Oppenlander ME, Maulucci CM, Sharan
AD, Heller J, Harrop JS, Jallo J, Prasad S. Intraoperative vancomycin use in spinal surgery:
single institution experience and microbial trends. Spine (Phila Pa 1976). 2014 Apr
1;39(7):550-5. doi: 10.1097/BRS.0000000000000241. PubMed PMID: 24480966. View
Abstract at PubMed
Ziewacz JE, Berven SH, Mummaneni VP, Tu TH, Akinbo OC, Lyon R, Mummaneni PV. The
design, development, and implementation of a checklist for intraoperative neuromonitoring
changes. Neurosurg Focus. 2012 Nov;33(5):E11. doi: 10.3171/2012.9.FOCUS12263. PubMed
PMID: 23116091. View Abstract at PubMed
Malhotra NR, Shaffrey CI. Intraoperative electrophysiological monitoring in spine surgery.
Spine (Phila Pa 1976). 2010 Dec 1;35(25):2167-79. doi: 10.1097/BRS.0b013e3181f6f0d0.
Review. PubMed PMID: 21102290. View Abstract at PubMed
Kim HJ, Bridwell KH, Lenke LG, Park MS, Song KS, Piyaskulkaew C, Chuntarapas T. Patients
with proximal junctional kyphosis requiring revision surgery have higher postoperative lumbar
lordosis and larger sagittal balance corrections. Spine (Phila Pa 1976). 2014 Apr
20;39(9):E576-80. doi: 10.1097/BRS.0000000000000246. PubMed PMID: 24480958. View
Abstract at PubMed
Maruo K, Ha Y, Inoue S, Samuel S, Okada E, Hu SS, Deviren V, Burch S, William S, Ames CP,
Mummaneni PV, Chou D, Berven SH. Predictive factors for proximal junctional kyphosis in
long fusions to the sacrum in adult spinal deformity. Spine (Phila Pa 1976). 2013 Nov
1;38(23):E1469-76. doi: 10.1097/BRS.0b013e3182a51d43. PubMed PMID: 23921319. View
Abstract at PubMed
Radcliff KE, Kepler CK, Jakoi A, Sidhu GS, Rihn J, Vaccaro AR, Albert TJ, Hilibrand AS. Adjacent
segment disease in the lumbar spine following different treatment interventions. Spine J. 2013
Oct;13(10):1339-49. doi: 10.1016/j.spinee.2013.03.020. Epub 2013 Jun 15. Review. PubMed
PMID: 23773433. View Abstract at PubMed
Lee JH, Kim JU, Jang JS, Lee SH. Analysis of the incidence and risk factors for the progression
of proximal junctional kyphosis following surgical treatment for lumbar degenerative
kyphosis: minimum 2-year follow-up. Br J Neurosurg. 2014 Apr;28(2):252-8. doi:
10.3109/02688697.2013.835369. Epub 2013 Dec 9. PubMed PMID: 24313308.
View Abstract at PubMed

89
Adult spine self-scored self-assessment examination AAOS 2015

Question 90 of 100

A B

Figures 90a and 90b are MR images of a 34-year-old man who is referred to
your office by his primary care physician after failing 4 months of nonsurgical
treatment that included epidural steroids for severe right arm pain occurring
in a C6 distribution. He also has associated paresthesias in this region. The
patient is weak in elbow flexion and wrist extension. What are his likely
outcomes if he is treated with a posterior foraminotomy instead of anterior
cervical diskectomy and fusion (ACDF)?

1- Similar incidence of postsurgical neck pain with higher risk for radiculopathy
recurrence at the same level
2- Higher incidence of postsurgical neck pain and radiculopathy recurrence at
the same level
3- Higher incidence of postsurgical neck pain and adjacent-level radiculopathy
4- Lower incidence of adjacent segment degeneration and postsurgical neck
pain

PREFERRED RESPONSE: 2- Higher incidence of postsurgical neck pain and


radiculopathy recurrence at the same level

DISCUSSION

90
Adult spine self-scored self-assessment examination AAOS 2015

This patient has a right-sided C5-C6 disk herniation causing C6 radicular


symptoms in the right upper extremity. Studies have shown that both ACDF
and posterior foraminotomy confer similar results in terms of pain relief and
functional outcome. Patients treated with posterior foraminotomy are at
higher risk for neck pain and recurrence of radiculopathy at the same level.
Those who receive ACDF are at higher risk for occurrence of radiculopathy at
an adjacent level.

RECOMMENDED READINGS

Rao RD, Currier BL, Albert TJ, Bono CM, Marawar SV, Poelstra KA, Eck JC. Degenerative
cervical spondylosis: clinical syndromes, pathogenesis, and management. J Bone Joint Surg
Am. 2007 Jun;89(6):1360-78. Review. PubMed PMID: 17575617. View Abstract at PubMed
Bolesta MJ, Gill K. Acute neck pain and cervical disk herniation. In: Spivak JM, Connolly PJ,
eds. Orthopaedic Knowledge Update: Spine 3. Rosemont, IL: American Academy of
Orthopaedic Surgeons; 2006:227-234.

Question 91 of 100

A B C

Figures 91a through 91c are CT images of a 76-year-old man who was
involved in a motor vehicle collision. Which of the following scenarios would
pose a contraindication to closed reduction of this injury prior to MR imaging?

1- American Spinal Injury Association Impairment Scale Grade B neurologic


deficit

91
Adult spine self-scored self-assessment examination AAOS 2015

2- Nondisplaced frontal bone fracture


3- Obtunded status and a blood alcohol concentration higher than 0.2%
4- Presence of facet fractures

PREFERRED RESPONSE: 3- Obtunded status and a blood alcohol concentration


higher than 0.2%

DISCUSSION

This patient has bilateral jumped facet joints at C6-7. Although MR imaging is
useful for revealing disk herniations, cord injuries, and bony fragments, early
closed reduction to restore anatomic alignment may be attempted prior to MR
imaging because reduction will decrease pressure on the cord. There have
been reports of catastrophic outcomes with closed reduction in patients who
are intubated when disk fragments are pushed into the spinal cord.
Consequently, closed reduction should be attempted only in awake and
cooperative patients for whom neurologic status monitoring is possible. MR
imaging is generally performed after reduction is attempted (successful or
not).

RECOMMENDED READINGS

Initial closed reduction of cervical spine fracture-dislocation injuries. Neurosurgery. 2002


Mar;50(3 Suppl):S44-50. Review. PubMed PMID: 12431286. View Abstract at PubMed
Radcliff K, Sonagli MA, Delasotta L, Singh N, Morrison E, Levine AM, Vaccaro AR. Cervical
facet fractures and dislocations. In: Zigler JE, Eismont FJ, Garfin SR, Vaccaro AR, eds. Spine
Trauma. 2nd ed. Rosemont, IL: American Academy of Orthopaedic Surgeons; 2011:441-
464.
Vaccaro AR, Falatyn SP, Flanders AE, Balderston RA, Northrup BE, Cotler JM. Magnetic
resonance evaluation of the intervertebral disc, spinal ligaments, and spinal cord before and
after closed traction reduction of cervical spine dislocations. Spine (Phila Pa 1976). 1999 Jun
15;24(12):1210-7. PubMed PMID: 10382247. View Abstract at PubMed
Wimberley DW, Vaccaro AR, Goyal N, Harrop JS, Anderson DG, Albert TJ, Hilibrand AS. Acute
quadriplegia following closed traction reduction of a cervical facet dislocation in the setting of
ossification of the posterior longitudinal ligament: case report. Spine (Phila Pa 1976). 2005
Aug 1;30(15):E433-8. PubMed PMID: 16094262. View Abstract at PubMed

Question 92 of 100

92
Adult spine self-scored self-assessment examination AAOS 2015

Figures 92a through 92c are the radiographs of a 34-year-old man with low-
back pain and an inability to walk upright. What is the appropriate surgical
treatment?

A B C

1- Smith-Peterson osteotomies at T12-L1, L1-L2 and L2-3.


2- Vertebral column resection through a posterior approach
3- Anterior-posterior osteotomy
4- Pedicle subtraction osteotomy at L3

PREFERRED RESPONSE: 4- Pedicle subtraction osteotomy at L3

DISCUSSION

This patient has a marked fixed sagittal imbalance and a mild coronal
imbalance. His fused sacroiliac joints indicate ankylosing spondylitis. Sufficient
correction likely can be achieved with a pedicle subtraction osteotomy in the
midlumbar spine. Smith-Petersen osteotomies necessitate flexibility of the
anterior column, which is not associated with this diagnosis. Also, osteoclasis
can result in vascular injuries. Vertebral column resection should not be
needed in this case.

93
Adult spine self-scored self-assessment examination AAOS 2015

RECOMMENDED READINGS

Patel NM, Jenis LG. Inflammatory arthritis of the spine. In: Spivak JM, Connolly PJ, eds.
Orthopaedic Knowledge Update: Spine 3. Rosemont, IL: American Academy of Orthopaedic
Surgeons; 2006:339-349.
Kim KT, Suk KS, Cho YJ, Hong GP, Park BJ. Clinical outcome results of pedicle subtraction
osteotomy in ankylosing spondylitis with kyphotic deformity. Spine (Phila Pa 1976). 2002 Mar
15;27(6):612-8. PubMed PMID: 11884909. View Abstract at PubMed

Question 93 of 100

During the approach to the lumbar spine for an L4-L5 anterior lumbar
interbody fusion, which structure generally is found overlying the anterior
surface of the L4 vertebra?

1- Aorta
2- Right common iliac artery
3- Left common iliac vein
4- Right ureter

PREFERRED RESPONSE: 1- Aorta

DISCUSSION

During an anterior approach to the L4-L5 disk space for anterior lumbar
interbody fusion, meticulous exposure is paramount to allow for safe
preparation of the disk space and subsequent arthrodesis. Although all of
these structures can come into play during the exposure, the aorta lies
anterior to the L4 vertebral body and bifurcates at this level. The vena cava
bifurcates just distal to this. The ureters lie to both sides of the anterior spine.
The right common iliac artery and the left common iliac vein originate after
the bifurcation of the great vessels and lie caudal to the L4 vertebra.

RECOMMENDED READINGS

Hoppenfeld S, deBoer P: Surgical Exposures in Orthopaedics: The Anatomic Approach. 3rd


ed. Philadelphia, PA: Lippincott Williams & Wilkins; 2003:226-235.
Agur AMR, Lee MJ, eds. Grant's Atlas of Anatomy. 10th ed. Philadelphia, PA: Lippincott
Williams & Wilkins; 1999:89.

94
Adult spine self-scored self-assessment examination AAOS 2015

Question 94 of 100

What are the most likely examination findings of the patient with the images
shown in Figures 94a and 94b?

A B

1- Diminished sensation over the distal anterior thigh and medial leg with
quadriceps and anterior tibialis weakness and a diminished patellar tendon
reflex on the left
2- Diminished sensation over the posterior leg, lateral leg, and plantar foot
with weakness of plantar flexion and a diminished Achilles tendon reflex on
the right
3- Diminished sensation over the lateral leg and dorsal foot with anterior
tibialis and extensor hallucis longus and anterior tibialis weakness on the left
4- Diminished sensation over the lateral leg and dorsal foot with anterior
tibialis and extensor hallucis longus and anterior tibialis weakness on the
right

PREFERRED RESPONSE: 4- Diminished sensation over the lateral leg and


dorsal foot with anterior tibialis and extensor hallucis longus and anterior
tibialis weakness on the right

95
Adult spine self-scored self-assessment examination AAOS 2015

DISCUSSION

The findings on MR imaging reveal a right-sided L4-L5 disk extrusion with


cephalad migration of the disk fragment. The axial image shows marked
displacement of the traversing right L5 nerve root. The physical findings noted
in Response 4 above are typical of a right L5 sensory and motor radiculopathy
that would be associated with this level of disk extrusion. Although an
extrusion at this level can affect the exiting L4 nerve root resulting in an L4
radiculopathy as described in Response 1, the findings described in this
response are contralateral to the disk herniation and not likely to be present.
The other responses describe findings associated with left and right S1
radiculopathy, which more typically are associated with an L5-S1 disk
herniation/extrusion

RECOMMENDED READINGS

Hoppenfeld S: Orthopaedic Neurology: A Diagnostic Guide to Neurologic Levels. Philadelphia,


PA, JB Lippincott, 1977, pp 7-49.
Haak MH. History and physical examination. In: Spivak JM, Connolly PJ, eds. Orthopaedic
Knowledge Update: Spine 3. Rosemont, IL: American Academy of Orthopaedic Surgeons;
2006:43-55.

Question 95 of 100

A 69-year-old patient with diabetes has had acute-onset back pain and
difficulty with ambulation for several hours. Evaluation reveals a temperature
of 38.3°C, a white blood cell (WBC) count of 14000/µL (reference range [rr],
4500-11000/µL), C-reactive protein (CRP) level of 120 mg/L (rr, 0.08-3.1
mg/L), erythrocyte sedimentation rate of 130 mm/h (rr, 0-20 mm/h), normal
rectal examination findings, and normal sensation to light touch. Motor
function testing of the lower extremities reveals 3/5 ankle dorsiflexion and 4/5
plantar flexion strength bilaterally. An MR image reveals a large epidural
abscess from L1-5. What is the most appropriate treatment at this time?

1- Medical management with intravenous (IV) antibiotics and observation


2- CT-guided aspiration of the abscess before initiating antibiotics
3- Surgical decompression and IV antibiotics
4- Blood cultures and re-evaluation in 24 hours

96
Adult spine self-scored self-assessment examination AAOS 2015

PREFERRED RESPONSE: 3- Surgical decompression and IV antibiotics

DISCUSSION

Epidural abscess is a serious and potentially disastrous condition. Although


medical management is effective in some situations, surgical decompression
is considered urgent with the presence of a neurological deficit. Medical
management can be considered in the case of a neurologically intact patient,
particularly when the microorganism has been identified. If medical
management is chosen, careful observation and serial examination for
neurologic deterioration is required. Surgical decompression is indicated if a
patient's neurologic status worsens or if medical management failure is noted.
Additionally, diabetes, a CRP level higher than 115 mg/L, WBC higher than
12500/µL , and bacteremia have proven predictive of medical treatment
failure. This patient would be a better candidate for urgent surgical
decompression and subsequent IV antibiotics than for medical management.

RECOMMENDED READINGS

Patel AR, Alton TB, Bransford RJ, Lee MJ, Bellabarba CB, Chapman JR. Spinal epidural
abscesses: risk factors, medical versus surgical management, a retrospective review of 128
cases. Spine J. 2014 Feb 1;14(2):326-30. doi: 10.1016/j.spinee.2013.10.046. Epub 2013
Nov 12. PubMed PMID: 24231778.View Abstract at PubMed
Kim SD, Melikian R, Ju KL, Zurakowski D, Wood KB, Bono CM, Harris MB. Independent
predictors of failure of nonoperative management of spinal epidural abscesses. Spine J. 2014
Aug 1;14(8):1673-9. doi: 10.1016/j.spinee.2013.10.011. Epub 2013 Oct 30. PubMed PMID:
24373683. View Abstract at PubMed

CLINICAL SITUATION FOR QUESTIONS 96 AND 97

Figures 96a and 96b are the CT scans of a 32-year-old man who was thrown
from his motorcycle. He has humeral shaft and femoral shaft fractures. A
secondary survey reveals substantial tenderness to his lower thoracic spine.
He is awake and alert and his movement is limited by pain secondary to the
extremity fractures. He is otherwise neurologically intact.

97
Adult spine self-scored self-assessment examination AAOS 2015

A B

Question 96 of 100

Which factor is most important when making a decision regarding surgery with
this patient?

1- Degree of kyphosis
2- Mechanism of injury
3- The patient's other injuries
4- The patient's bone quality

PREFERRED RESPONSE: 3- The patient's other injuries

Question 97 of 100

If the patient had an isolated spine injury without neurologic deficit, the most
appropriate next step would be

1- anterior corpectomy with percutaneous pedicle screw stabilization.


2- percutaneous pedicle screw stabilization.
3- posterior pedicle screw stabilization with fusion.
4- MRI.

98
Adult spine self-scored self-assessment examination AAOS 2015

PREFERRED RESPONSE: 4- MRI.

DISCUSSION

The treatment of thoracolumbar burst fractures has evolved over the years.
In the absence of a neurologic deficit or a posterior ligamentous complex
injury, nonsurgical treatment is as effective as surgery. The degree of spinal
canal compromise is not a risk factor for neurologic symptoms. Similarly,
although kyphosis may be a marker of more significant injury, the degree of
kyphosis does not correlate with chronic pain. In the setting of a burst
fracture, MRI can be used to evaluate the integrity of the posterior
ligamentous complex. Polytrauma may be considered a relative indication for
surgical intervention in the setting of a stable burst fracture.

RECOMMENDED READINGS

Rechtine GR 2nd. Nonoperative management and treatment of spinal injuries. Spine (Phila Pa
1976). 2006 May 15;31(11 Suppl):S22-7; discussion S36. Review. PubMed PMID: 16685232.
View Abstract at PubMed
Shen WJ, Shen YS. Nonsurgical treatment of three-column thoracolumbar junction burst
fractures without neurologic deficit. Spine (Phila Pa 1976). 1999 Feb 15;24(4):412-5. PubMed
PMID: 10065527. View Abstract at PubMed
Wood K, Buttermann G, Mehbod A, Garvey T, Jhanjee R, Sechriest V. Operative compared
with nonoperative treatment of a thoracolumbar burst fracture without neurological deficit. A
prospective, randomized study. J Bone Joint Surg Am. 2003 May;85-A(5):773-81. Erratum
in: J Bone Joint Surg Am. 2004 Jun;86-A(6):1283. Butterman, G [corrected to Buttermann,
G]. PubMed PMID: 12728024. View Abstract at PubMed
Wood KB, Li W, Lebl DS, Ploumis A. Management of thoracolumbar spine fractures. Spine J.
2014 Jan;14(1):145-64. doi: 10.1016/j.spinee.2012.10.041. Review. PubMed PMID:
24332321.View Abstract at PubMed

Question 98 of 100

Surgical restoration of sagittal balance of an adult spinal deformity will have


which effect on outcome?

1- No effect
2- Improve leg-related symptoms but not back pain
3- Improve quality of life and back pain
4- Improve quality of life and leg-related symptoms

99
Adult spine self-scored self-assessment examination AAOS 2015

PREFERRED RESPONSE: 3- Improve quality of life and back pain

DISCUSSION

The influence of sagittal balance on outcomes following fusion-based


procedures for degenerative conditions of the lumbar spine has only recently
been appreciated. Restoration of sagittal spinal balance improves low-back-
pain outcomes and quality of life. Sagittal spinal balance has not been shown
to relieve neurogenic claudication attributable to spinal stenosis.

RECOMMENDED READINGS

Li Y, Hresko MT. Radiographic analysis of spondylolisthesis and sagittal spinopelvic deformity.


J Am Acad Orthop Surg. 2012 Apr;20(4):194-205. doi: 10.5435/JAAOS-20-04-194. Review.
PubMed PMID: 22474089. View Abstract at PubMed
Korovessis P, Repantis T, Papazisis Z, Iliopoulos P. Effect of sagittal spinal balance, levels of
posterior instrumentation, and length of follow-up on low back pain in patients undergoing
posterior decompression and instrumented fusion for degenerative lumbar spine disease: a
multifactorial analysis. Spine (Phila Pa 1976). 2010 Apr 15;35(8):898-905. doi:
10.1097/BRS.0b013e3181d51e84. PubMed PMID: 20354466. View Abstract at PubMed

CLINICAL SITUATION FOR QUESTIONS 99 AND 100

Figures 99a and 99b are MR images of a 59-year-old man with a history of
intravenous (IV) drug abuse who arrives at the emergency department with
malaise and fever. Upon admission, the patient's temperature is 38.9°C, his
white blood cell count is 17000/µL (reference range [rr], 4500-11000/µL), his
erythrocyte sedimentation rate is 98 mm/h (rr, 0-20 mm/h), and his C-
reactive protein level is 45 mg/L (rr, 0.08-3.1 mg/L). He is admitted to the
medical service to evaluate the source of his fevers. On hospital day 1, the
patient reports weakness in his left arm and leg. Blood cultures are positive
for methicillin-resistant Staphylococcus aureus.

100
Adult spine self-scored self-assessment examination AAOS 2015

A B

Question 99 of 100

What is the most likely diagnosis?

1- Central cord syndrome


2- Cerebrovascular stroke
3- Spinal epidural abscess
4- Guillain-Barre syndrome from IV drug abuse

PREFERRED RESPONSE: 3- Spinal epidural abscess

Question 100 of 100

What is the most appropriate treatment at this time?

1- IV steroids and antibiotics


2- IV antibiotics
3- Anterior decompression and fusion
4- Posterior laminectomy and instrumented fusion

PREFERRED RESPONSE: 3- Anterior decompression and fusion

DISCUSSION

101
Adult spine self-scored self-assessment examination AAOS 2015

Prompt diagnosis and treatment of patients with spinal epidural abscess is


crucial to maintain and/or improve neurologic function. This clinical scenario
stresses the importance of advanced imaging studies. It is also important to
recognize the imaging features of spinal epidural abscess. T1-weighted
gadolinium-enhanced images show ring enhancement with a central
nonenhancing, low-signal area. In such a case, urgent decompression is
indicated. Because of the location of the abscess, which is anterior to the
spinal cord, an anterior decompression and reconstruction (ie, fusion) is
probably the best treatment plan. Steroids are contraindicated in the presence
of an epidural abscess. IV antibiotics alone will not adequately treat a patient
with a neurological deficit. A posterior laminectomy and fusion will not safely
allow access to the abscess.

RECOMMENDED READINGS

Bluman EM, Palumbo MA, Lucas PR. Spinal epidural abscess in adults. J Am Acad Orthop Surg.
2004 May-Jun;12(3):155-63. Review. PubMed PMID: 15161168. View Abstract at PubMed
Ghobrial GM, Beygi S, Viereck MJ, Maulucci CM, Sharan A, Heller J, Jallo J, Prasad S, Harrop
JS. Timing in the surgical evacuation of spinal epidural abscesses. Neurosurg Focus. 2014
Aug;37(2):E1. doi: 10.3171/2014.6.FOCUS14120. PubMed PMID: 25081958. View Abstract
at PubMed

This is the last question of the exam.

102

You might also like